You are on page 1of 34

Clinical Microscopy 1 Test Review and Case Studies

Jethro Rada Jr.


Chapter 1 A. Vectors
1. In the urinalysis laboratory the primary source in the B. Sharps contamination
chain of infection would be: C. Aerosols
A. Patients D. Specimen contamination
B. Needlesticks 11. An employee who accidentally spills acid on his arm
C. Specimens should immediately:
D. Biohazardous waste A. Neutralize the acid with a base
2. The best way to break the chain of infection is: B. Hold the arm under running water for 15 minutes
A. Hand sanitizing C. Consult the MSDSs
B. Personal protective equipment D. Wrap the arm in gauze and go to the emergency
C. Aerosol prevention department
D. Decontamination 12. When combining acid and water, ensure that:
3. The current routine infection control policy developed A. Acid is added to water
by CDC and followed in all health-care settings is: B. Water is added to acid
A. Universal Precautions C. They are added simultaneously
B. Isolation Precautions D. Water is slowly added to acid
C. Blood and Body Fluid Precautions 13. An employee can learn the carcinogenic potential of
D. Standard Precautions potassium chloride by consulting the:
4. An employee who is accidentally exposed to a possible A. Chemical hygiene plan
blood-borne pathogen should immediately: B. Material safety data sheets
A. Report to a supervisor C. OSHA standards
B. Flush the area with water D. Urinalysis procedure manual
C. Clean the area with disinfectant 14. Employees should not work with radioisotopes if
D. Receive HIV prophylaxis they are:
5. Personnel in the urinalysis laboratory should wear lab A. Wearing contact lenses
coats that: B. Allergic to iodine
A. Do not have buttons C. Sensitive to latex
B. Are fluid-resistant D. Pregnant
C. Have short sleeves 15. All of the following are safe to do when removing the
D. Have full-length zippers source of an electric shock except:
6. All of the following should be discarded in biohazardous A. Pulling the person away from the instrument
waste containers except: B. Turning off the circuit breaker
A. Urine specimen containers C. Using a glass container to move the instrument
B. Towels used for decontamination D. Unplugging the instrument
C. Disposable lab coats 16. The acronym PASS refers to:
D. Blood collection tubes A. Presence of vital chemicals
7. An employer who fails to provide sufficient gloves for B. Operation of a fire extinguisher
the employees may be fined by the: C. Labeling of hazardous material
A. CDC D. Presence of radioactive substances
B. NFPA 17. The system used by firefighters when a fire occurs in the
C. OSHA laboratory is:
D. FDA A. MSDS
8. An acceptable disinfectant for blood and body fluid B. RACE
decontamination is: C. NFPA
A. Sodium hydroxide D. PASS
B. Antimicrobial soap 18. A class ABC fire extinguisher contains:
C. Hydrogen peroxide A. Sand
D. Sodium hypochlorite B. Water
9. Proper handwashing includes all of the following C. Dry chemicals
except: D. Acid
A. Using warm water 19. The first thing to do when a fire is discovered is to:
B. Rubbing to create a lather A. Rescue persons in danger
C. Rinsing hands in a downward position B. Activate the alarm system
D. Turning on the water with a paper towel C. Close doors to other areas
10. Centrifuging an uncapped specimen may produce a D. Extinguish the fire if possible
biologic hazard in the form of: 20. If a red rash is observed after removing gloves, the
employee: _____ Calibrating the centrifuge
A. May be washing her hands too often _____ Collecting a timed urine specimen
B. May have developed a latex allergy 30. The testing of sample from an outside agency and the
C. Should apply cortisone cream comparison
D. Should not rub the hands so vigorously of results with participating laboratories is called:
21. Pipetting by mouth is: A. External QC
A. Acceptable for urine but not serum B. Electronic QC
B. Not acceptable without proper training C. Internal QC
C. Acceptable for reagents but not specimens D. Proficiency testing
D. Not acceptable in the laboratory 31. A color change that indicates that a sufficient amount of
22. The NPFA classification symbol contains information on patient’s specimen or reagent is added correctly to the
all of the following except: test system would be an example of:
A. Fire hazards A. External QC
B. Biohazards B. Equivalent QC
C. Reactivity C. Internal QC
D. Health hazards D. Proficiency testing
23. The classification of a fire that can be extinguished with 32. What steps are taken when the results of reagent strip
water is: QC are outside of the stated confidence limits?
A. Class A A. Check the expiration date of the reagent strip
B. Class B B. Run a new control
C. Class C C. Open a new reagent strips container
D. Class D D. All of the above
24. Employers are required to provide free immunization for: 33. When a new bottle of QC material is opened, what
A. HIV information is placed on the label?
B. HTLV-1 A. The supervisor’s initials
C. HBV B. The lot number
D. HCV C. The date and the laboratory worker’s initials
25. A possible physical hazard in the hospital is: D. The time the bottle was opened
A. Wearing closed-toed shoes 34. When a control is run, what information is
B. Not wearing jewelry documented?
C. Having short hair A. The lot number
D. Running to answer the telephone B. Expiration date of the control
26. Quality assessment refers to: C. The test results
A. Analysis of testing controls D. All of the above
B. Increased productivity Case Study
C. Precise control results 1. State a possible reason for an accreditation team to
D. Quality of specimens and patient care
report a deficiency in the following situations:
27. During laboratory accreditation inspections, procedure
a. The urine microscopic reporting procedure has
manuals are examined for the presence of:
been recently revised.
A. Critical values
b. An unusually high number of urine specimens are
B. Procedure references
being rejected because of improper collection.
C. Procedures for specimen preservation
c. A key statement is missing from the Clinitest
D. All of the above
procedure.
28. As supervisor of the urinalysis laboratory, you have just
d. Open control bottles in the refrigerator are examined.
adopted a new procedure. You should:
2. As the new supervisor of the urinalysis section, you
A. Put the package insert in the procedure manual
encounter the following situations. Explain whether
B. Put a complete, referenced procedure in the manual
you would accept them or take corrective action.
C. Notify the microbiology department
a. You are told that the supervisor always performs
D. Put a cost analysis study in the procedure manual
the CAP proficiency survey.
29. Indicate whether each of the following would be
b. QC is not performed daily on the reagent strips.
considered
c. The urinalysis section is primarily staffed by
a 1) preexamination, 2) examination, or 3) postexamination
personnel assigned to other departments for whom
factor by placing the appropriate number in
you have no personnel data.
the blank:
3. The medical technologist was assigned to test 10 urine
_____ Reagent expiration date
specimens chemically. She removed 10 strips from the
_____ Rejecting a contaminated specimen
container and proceeded with testing. Several patients’
_____ Constructing a Levy-Jennings chart
urine indicated a trace positive glucose in the urine.
_____ Telephoning a positive Clinitest result on a
She then opened a new bottle of reagent dipsticks and
newborn
proceeded to perform the QC. The negative control B. Specimen contains toilet paper
also tested as a trace positive for glucose. The medical C. Label and requisition do not match
technologist consulted the supervisor. The supervisor D. Outside of the container has fecal material
ran the QC and the results were in the correct range. contamination
After observing the medical technologist’s technique, 9. A cloudy specimen received in the laboratory may have
the supervisor realized that the medical technologist been preserved using:
was waiting too long to read the glucose results and A. Boric acid
therefore reporting erroneous results. B. Chloroform
a. What is wrong with this scenario? C. Refrigeration
b. Who should run the QC for each test? Why? D. Formalin
c. When should controls be run? 10. For general screening the most frequently collected
d. What do you do when the QC is out of range? specimen is a:
e. When can you report patient results? A. Random one
B. First morning
Chapter 2 C. Midstream clean-catch
D. Timed
1. The average daily output of urine is:
11. The primary advantage of a first morning specimen over
A. 200 mL
a random specimen is that it:
B. 500 mL
A. Is less contaminated
C. 1200 mL
B. Is more concentrated
D. 2500 mL
C. Is less concentrated
2. An unidentified fluid is received in the laboratory with a
D. Has a higher volume
request to determine whether the fluid is urine or another
12. If a routine urinalysis and a culture are requested on a
body fluid. Using routine laboratory tests, what tests
catheterized specimen, then:
would determine that the fluid is most probably urine?
A. Two separate containers must be collected
A. Glucose and ketones
B. The routine urinalysis is performed first
B. Urea and creatinine
C. The patient must be recatheterized
C. Uric acid and amino acids
D. The culture is performed first
D. Protein and amino acids
13. If a patient fails to discard the first specimen when
3. The primary inorganic substance found in urine is:
collecting a timed specimen the:
A. Sodium
A. Specimen must be recollected
B. Phosphate
B. Results will be falsely elevated
C. Chloride
C. Results will be falsely decreased
D. Calcium
D. Both A and B
4. A patient presenting with polyuria, nocturia, polydipsia,
14. The primary cause of unsatisfactory results in an
and a low urine specific gravity is exhibiting symptoms of:
unpreserved
A. Diabetes insipidus
routine specimen not tested for 8 hours is:
B. Diabetes mellitus
A. Bacterial growth
C. Urinary tract infection
B. Glycolysis
D. Uremia
C. Decreased pH
5. A patient with oliguria might progress to having:
D. Chemical oxidation
A. Nocturia
15. Prolonged exposure of a preserved urine specimen to
B. Polyuria
light will cause:
C. Polydipsia
A. Decreased glucose
D. Anuria
B. Increased cells and casts
6. All of the following are characteristics of recommended
C. Decreased bilirubin
urine containers except:
D. Increased bacteria
A. A flat bottom
16. Which of the following would be least affected in a
B. A capacity of 50 mL
specimen that has remained unpreserved at room
C. A snap-on lid
temperature for more than 2 hours?
D. Are disposable
A. Urobilinogen
7. Labels for urine containers are:
B. Ketones
A. Attached to the container
C. Protein
B. Attached to the lid
D. Nitrite
C. Placed on the container prior to collection
17. Bacterial growth in an unpreserved specimen will:
D. Not detachable
A. Decrease clarity
8. A urine specimen may be rejected by the laboratory for
B. Increase bilirubin
all of the following reasons except the fact that the:
C. Decrease pH
A. Requisition states the specimen is catheterized
D. Increase glucose
18. The most sterile specimen collected is a: 2. The function of the peritubular capillaries is:
A. Catheterized A. Reabsorption
B. Midstream clean-catch B. Filtration
C. Three-glass C. Secretion
D. Suprapubic aspiration D. Both A and C
d. How can this defense be avoided? Clinical Sit 3. Blood flows through the nephron in the following order:
19. Which of the following would not be given to a patient A. Efferent arteriole, peritubular capillaries, vasa recta,
prior to the collection of a midstream clean-catch specimen? afferent arteriole
A. Sterile container B. Peritubular capillaries, afferent arteriole, vasa recta,
B. Iodine cleanser efferent arteriole
C. Antiseptic towelette C. Afferent arteriole, peritubular capillaries, vasa recta,
D. Instructions efferent arteriole
20. Urine specimen collection for drug testing requires the D. Efferent arteriole, vasa recta, peritubular capillaries,
collector to do all of the following except: afferent arteriole
A. Inspect the specimen color 4. Filtration of protein is prevented in the glomerulus by:
B. Perform reagent strip testing A. Hydrostatic pressure
C. Read the specimen temperature B. Oncotic pressure
D. Fill out a chain-of-custody form C. Renin
Case Study D. The glomerular filtration barrier
1. A patient brings a first morning specimen to the 5. The renin-angiotensin-aldosterone system is responsible
laboratory at 1:00 p.m. for
a. How could this affect the urinalysis results? all of the following except:
b. What could the patient say that would make the A. Vasoconstriction of the afferent arteriole
specimen satisfactory for testing? B. Vasoconstriction of the efferent arteriole
2. A patient collecting a midstream clean-catch specimen C. Reabsorbing sodium
voids immediately into the container. D. Releasing aldosterone
a. How could this affect the clarity of the specimen? 6. The primary chemical affected by the renin-
b. How could this affect the microscopic examination? angiotensinaldosterone
3. A patient brings a 24-hour timed specimen to the system is:
laboratory and reports that he or she forgot to collect A. Chloride
a specimen voided during the night. B. Sodium
a. How will this affect the results of a quantitative test C. Potassium
for creatinine? D. Hydrogen
b. What should the patient be told to do? 7. Secretion of renin is stimulated by:
4. You receive a urine preservative tube for culture A. Juxtaglomerular cells
containing a volume of specimen that is considerably B. Angiotensin I and II
below the minimum fill line. C. Macula densa cells
a. Could this affect the culture? D. Circulating angiotensin-converting enzyme
b. Why? 8. The hormone aldosterone is responsible for:
5. A worker suspects that he or she will be requested to A. Hydrogen ion secretion
collect an unwitnessed urine specimen for drug analysis. B. Potassium secretion
He or she carries a substitute specimen in his or her C. Chloride retention
pocket for 2 days before being told to collect the specimen. D. Sodium retention
Shortly after the worker delivers the specimen to 9. The fluid leaving the glomerulus has a specific
the collector, he or she is instructed to collect another gravity of:
specimen. A. 1.005
a. What test was performed on the specimen to B. 1.010
determine possible specimen manipulation? C. 1.015
b. How was the specimen in this situation affected? D. 1.020
c. If a specimen for drug analysis tests positive, state a 10. For active transport to occur, a chemical:
possible defense related to specimen collection and A. Must combine with a carrier protein to create
handling that an attorney might employ. electrochemical energy
B. Must be filtered through the proximal convoluted
Chapter 3 tubule
C. Must be in higher concentration in the filtrate than
1. The type of nephron responsible for renal concentration
in the blood
is the:
D. Must be in higher concentration in the blood than in
A. Cortical
the filtrate
B. Juxtaglomerular
11. Which of the tubules is impermeable to water?
A. Proximal convoluted tubule 21. Clearance tests used to determine the glomerular
B. Descending loop of Henle filtration
C. Ascending loop of Henle rate must measure substances that are:
D. Distal convoluted tubule A. Not filtered by the glomerulus
12. Glucose will appear in the urine when the: B. Completely reabsorbed by the proximal convoluted
A. Blood level of glucose is 200 mg/dL tubule
B. Tm for glucose is reached C. Secreted in the distal convoluted tubule
C. Renal threshold for glucose is exceeded D. Neither reabsorbed or secreted by the tubules
D. All of the above Chapter 3 | Renal Function 55
13. Concentration of the tubular filtrate by the 22. Performing a clearance test using radionucleotides:
countercurrent A. Eliminates the need to collect urine
mechanism depends on all of the following B. Does not require an infusion
except: C. Provides visualization of the filtration
A. High salt concentration in the medulla D. Both A and C
B. Water-impermeable walls of the ascending loop of 23. Variables that are included in the MDRD-IDSM estimated
Henle creatinine clearance calculations include all of the
C. Reabsorption of sodium and chloride from the following except:
ascending loop of Henle A. Serum creatinine
D. Reabsorption of water in the descending loop of B. Weight
Henle C. Age
14. ADH regulates the final urine concentration by D. Gender
controlling: 24. An advantage to using cystatin C to monitor GFR is that:
A. Active reabsorption of sodium A. It does not require urine collection
B. Tubular permeability B. It is not secreted by the tubules
C. Passive reabsorption of urea C. It can be measured by immunoassay
D. Passive reabsorption of chloride D. All of the above
15. Decreased production of ADH: 25. Solute dissolved in solvent will:
A. Produces a low urine volume A. Raise the vapor pressure
B. Produces a high urine volume B. Lower the boiling point
C. Increases ammonia excretion C. Decrease the osmotic pressure
D. Affects active transport of sodium D. Lower the freezing point
16. Bicarbonate ions filtered by the glomerulus are returned 26. Substances that may interfere with freezing point
to the blood: measurement
A. In the proximal convoluted tubule of urine and serum osmolarity include all of the
B. Combined with hydrogen ions following except:
C. By tubular secretion A. Ethanol
D. All of the above B. Lactic acid
17. If ammonia is not produced by the distal convoluted C. Sodium
tubule, the urine pH will be: D. Lipids
A. Acidic 27. Clinical osmometers use NaCl as a reference solution
B. Basic because:
18. Place the appropriate letter in front of the following A. 1 g molecular weight of NaCl will lower the freezing
clearance point 1.86oC
substances: B. NaCl is readily frozen
A. Exogenous C. NaCl is partially ionized similar to the composition
B. Endogenous of urine
____ beta2-microglobulin D. 1 g equivalent weight of NaCl will raise the freezing
____ creatinine point 1.86oC
____ cystatin C 28. The normal serum osmolarity is:
____ 125I-iothalmate A. 50 to 100 mOsm
19. The largest source of error in creatinine clearance tests is: B. 275 to 300 mOsm
A. Secretion of creatinine C. 400 to 500 mOsm
B. Improperly timed urine specimens D. 3 times the urine osmolarity
C. Refrigeration of the urine 29. After controlled fluid intake, the urine-to-serum
D. Time of collecting blood sample osmolarity
20. Given the following information, calculate the creatinine ratio should be at least:
clearance: A. 1:1
24-hour urine volume: 1000 mL; serum creatinine: B. 2:1
2.0 mg/dL; urine creatinine: 200 mg/dL C. 3:1
D. 4:1 Explain your answer.
30. Calculate the free water clearance from the following 4. A laboratory is obtaining erratic serum osmolarity results
results: on a patient who is being monitored at 6 a.m., 12 p.m.,
urine volume in 6 hours: 720 mL; urine osmolarity: 6 p.m., and 12 a.m. Osmolarities are not performed on
225 mOsm; plasma osmolarity: 300 mOsm the night shift; therefore, the midnight specimen is run at
31. To provide an accurate measure of renal blood flow, a the same time as the 6 a.m. specimen.
test a. What two reasons could account for these
substance should be completely: discrepancies?
A. Filtered by the glomerulus b. What substance is causing the erratic results?
B. Reabsorbed by the tubules c. If a friend were secretly bringing the patient a pint of
C. Secreted when it reaches the distal convoluted whiskey every night, would this affect the
tubule results? Explain your answer.
D. Cleared on each contact with functional renal 5. Following overnight (6 p.m. to 8 a.m.) fluid deprivation,
tissue the urine-to-serum osmolarity ratio in a patient who is
32. Given the following data, calculate the effective renal exhibiting polyuria and polydipsia is 1:1. The ratio remains
plasma flow: the same when a second specimen is tested at
urine volume in 2 hours: 240 mL; urine PAH: 150 mg/dL; 10 a.m. ADH is then administered subcutaneously to
plasma PAH: 0.5 mg/dL the patient, and the fluid deprivation is continued until
33. Renal tubular acidosis can be caused by the: 2 p.m., when another specimen is tested.
A. Production of excessively acidic urine due to a. What disorder do these symptoms and initial laboratory
increased filtration of hydrogen ions results indicate?
B. Production of excessively acidic urine due to b. If the urine-to-serum osmolarity ratio on the 2 p.m.
increased secretion of hydrogen ions specimen is 3:1, what is the underlying cause of the
C. Inability to produce an acidic urine due to impaired patient’s disorder?
production of ammonia c. If the urine-to-serum osmolarity ratio on the 2 p.m.
D. Inability to produce an acidic urine due to increased specimen remains 1:1, what is the underlying cause of
production of ammonia the patient’s disorder?
34. Tests performed to detect renal tubular acidosis after
administering an ammonium chloride load include all Chapter 4
of the following except: 1. The concentration of a normal urine specimen can be
A. Urine ammonia estimated by which of the following?
B. Arterial pH A. Color
C. Urine pH B. Clarity
D. Titratable acidity C. Foam
Case Study D. Odor
1. A 44-year-old man diagnosed with acute tubular necrosis 2. The normal yellow color of urine is produced by:
has a blood urea nitrogen of 60 mg/dL and a blood A. Bilirubin
glucose level of 100 mg/dL. A 2+ urine glucose is also B. Hemoglobin
reported. C. Urobilinogen
a. State the renal threshold for glucose. D. Urochrome
b. What is the significance of the positive urine 3. The presence of bilirubin in a urine specimen produces a:
glucose and normal blood glucose? A. Yellow foam when shaken
2. A patient develops a sudden drop in blood pressure. B. White foam when shaken
a. Diagram the reactions that take place to ensure C. Cloudy specimen
adequate blood pressure within the nephrons. D. Yellow-red specimen
b. How do these reactions increase blood volume? 4. A urine specimen containing melanin will appear:
c. When blood pressure returns to normal, how does the A. Pale pink
kidney respond? B. Dark yellow
3. A physician would like to prescribe a nephrotoxic antibiotic C. Blue-green
for a 60-year-old Caucasian man. The patient has a D. Black
serum creatinine level of 1.5 mg/dL. 5. Specimens that contain hemoglobin can be visually
a. How can the physician determine whether it is safe to distinguished
prescribe this medication before the patient leaves the from those that contain RBCs because:
office? A. Hemoglobin produces a clear, yellow specimen
b. State two additional blood tests that the physician B. Hemoglobin produces a cloudy pink specimen
could use to continue monitoring this patient. C. RBCs produce a cloudy red specimen
c. If the patient has a history of prostate malignancy, D. RBCs produce a clear red specimen
would both of these methods provide reliable results? 6. A patient with a viscous orange specimen may have been:
A. Treated for a urinary tract infection 15. A specimen with a specific gravity of 1.035 would be
B. Taking vitamin B pills considered:
C. Eating fresh carrots A. Isosthenuric
D. Taking antidepressants B. Hyposthenuric
Study Questions C. Hypersthenuric
Chapter 4 | Physical Examination of Urine 67 D. Not urine
68 Part Two | Urinalysis 16. A specimen with a specific gravity of 1.001 would be
7. The presence of a pink precipitate in a refrigerated considered:
specimen A. Hyposthenuric
is caused by: B. Not urine
A. Hemoglobin C. Hypersthenuric
B. Urobilin D. Isosthenuric
C. Uroerythrin 17. A strong odor of ammonia in a urine specimen could
D. Beets indicate:
8. Microscopic examination of a clear urine that produces a A. Ketones
white precipitate after refrigeration will show: B. Normalcy
A. Amorphous urates C. Phenylketonuria
B. Porphyrins D. An old specimen
C. Amorphous phosphates 18. The microscopic of a clear red urine is reported as many
D. Yeast WBCs and epithelial cells. What does this suggest?
9. The color of urine containing porphyrins will be: A. Urinary tract infection
A. Yellow-brown B. Dilute random specimen
B. Green C. Hematuria
C. Orange D. Possible mix-up of specimen and sediment
D. Port wine 19. Which of the following would contribute the most to a
10. Which of the following specific gravities would be most urine osmolality?
likely to correlate with a pale yellow urine? A. One osmole of glucose
A. 1.005 B. One osmole of urea
B. 1.010 C. One osmole of sodium chloride
C. 1.020 D. All contribute equally
D. 1.030 20. Which of the following colligative properties is not stated
11. A urine specific gravity measured by refractometer is correctly?
1.029, and the temperature of the urine is 14°C. The specific A. The boiling pointing is raised by solute
gravity should be reported as: B. The freezing point is raised by solute
A. 1.023 C. The vapor pressure is lowered by solute
B. 1.027 D. The osmotic pressure is raised by solute
C. 1.029 Chapter 4 | Physical Examination of Urine 69
D. 1.032 21. An osmole contains:
12. The principle of refractive index is to compare: A. One gram molecular weight of solute dissolved in
A. Light velocity in solutions with light velocity in one liter of solvent
solids B. One gram molecular weight of solute dissolved
B. Light velocity in air with light velocity in solutions in one kilogram of solvent
C. Light scattering by air with light scattering by C. Two gram molecular weights of solute dissolved in
solutions one liter of solvent
D. Light scattering by particles in solution D. Two gram molecular weights of solute dissolved
13. A correlation exists between a specific gravity by in one kilogram of solvent
refractometer 22. The unit of osmolality measured in the clinical laboratory
of 1.050 and a: is the:
A. 2+ glucose A. Osmole
B. 2+ protein B. Milliosmole
C. First morning specimen C. Molecular weight
D. Radiographic dye infusion D. Ionic charge
14. A cloudy urine specimen turns black upon standing and 23. In the reagent strip specific gravity reaction the
has a specific gravity of 1.012. The major concern about polyelectrolyte:
this specimen would be: A. Combines with hydrogen ions in response to ion
A. Color concentration
B. Turbidity B. Releases hydrogen ions in response to ion
C. Specific gravity concentration
D. All of the above C. Releases hydrogen ions in response to pH
D. Combines with sodium ions in response to pH C. The sodium nitroprusside reaction
24. Which of the following will react in the reagent strip D. Diazo reactions
specific gravity test? 4. The reagent strip reaction that requires the longest
A. Glucose reaction
B. Radiographic dye time is the:
C. Protein A. Bilirubin
D. Chloride B. pH
Case Study C. Leukocyte esterase
1. Given the following physical urinalysis results, determine D. Glucose
additional urinalysis results that may be abnormal. 5. Quality control of reagent strips is performed:
a. A green specimen with a strong foul odor of ammonia A. Using positive and negative controls
b. A pale yellow urine with a specific gravity of 1.030 B. When results are questionable
c. A dark yellow specimen with yellow foam C. At least once every 24 hours
d. A cloudy red urine D. All of the above
2. The urology clinic questions a urinalysis report from the 6. All of the following are important to protect the integrity
laboratory. of reagent strips except:
The laboratory report states that a reagent strip reading A. Removing the desiccant from the bottle
of a specific gravity of 1.020, protein 3 g/dL, and glucose B. Storing in an opaque bottle
2 g/dL. The specific gravity in the urology clinic was C. Storing at room temperature
greater than 1.035. D. Resealing the bottle after removing a strip
a. Correct the refractometer reading to account for the 7. The principle of the reagent strip test for pH is the:
protein and glucose concentrations. What is the corrected A. Protein error of indicators
specific gravity? B. Greiss reaction
b. Do the specific gravities correlate? C. Dissociation of a polyelectrolyte
c. If the specific gravity was also checked using osmometry, D. Double indicator reaction
should the result agree with the laboratory or the 8. A urine specimen with a pH of 9.0:
urology clinic results? Why or why not? A. Indicates metabolic acidosis
3. State two pathologic causes of a clear red urine. B. Should be recollected
a. State a method that could distinguish between the two C. May contain calcium oxalate crystals
causes that does not require laboratory testing. D. Is seen after drinking cranberry juice
4. Mrs. Smith frequently shops at the farmer’s market near 9. In the laboratory, a primary consideration associated
her home. She notices her urine has a red color and with pH is:
brings a sample to her physician. The specimen tests A. Identifying urinary crystals
negative for blood. B. Monitoring vegetarian diets
a. What is a probable cause of Mrs. Smith’s red urine? C. Determining specimen acceptability
b. Mrs. Smith collects a specimen at the physician’s D. Both A and C
office. The color is yellow and the pH is 5.5. Is 10. Indicate the source of the following proteinurias by
this consistent with the previous answer? Why or placing
why not? a 1 for prerenal, 2 for renal, or 3 for postrenal in front
5. Is a clear urine always normal? Explain your answer. of the condition.
A. ____Microalbuminuria
B. ____Acute phase reactants
Part 5 C. ____Pre-eclampsia
1. Leaving excess urine on the reagent strip after removing D. ____Vaginal inflammation
it from the specimen will: E. ____Multiple myeloma
A. Cause run-over between reagent pads F. ____Orthostatic proteinuria
B. Alter the color of the specimen G. ____Prostatitis
C. Cause reagents to leach from the pads 11. The principle of the protein error of indicators reaction
D. Not affect the chemical reactions is that:
2. Failure to mix a specimen before inserting the reagent A. Protein keeps the pH of the urine constant
strip will primarily affect the: B. Albumin accepts hydrogen ions from the indicator
A. Glucose reading C. The indicator accepts hydrogen ions from
B. Blood reading albumin
C. Leukocyte reading D. Albumin changes the pH of the urine
D. Both B and C 12. All of the following will cause false-positive protein
3. Testing a refrigerated specimen that has not warmed to reagent strip values except:
room temperature will adversely affect: A. Microalbuminuria
A. Enzymatic reactions B. Highly buffered alkaline urines
B. Dye-binding reactions C. Delay in removing the reagent strip from the specimen
D. Contamination by quaternary ammonium compounds is to:
13. A patient with a 2+ protein reading in the afternoon is A. Estimate the glomerular filtration rate
asked B. Correct for hydration in random specimens
to submit a first morning specimen. The second specimen C. Avoid interference for alkaline urines
has a negative protein reading. This patient is: D. Correct for abnormally colored urines
A. Positive for orthostatic proteinuria 22. A patient with a normal blood glucose and a positive
B. Negative for orthostatic proteinuria urine glucose should be further checked for:
C. Positive for Bence Jones protein A. Diabetes mellitus
D. Negative for clinical proteinuria B. Renal disease
14. Testing for microalbuminuria is valuable for early C. Gestational diabetes
detection D. Pancreatitis
of kidney disease and monitoring patients with: 23. The principle of the reagent strip tests for glucose is the:
A. Hypertension A. Peroxidase activity of glucose
B. Diabetes mellitus B. Glucose oxidase reaction
C. Cardiovascular disease risk C. Double sequential enzyme reaction
D. All of the above D. Dye-binding of glucose and chromogen
15. The primary chemical on the reagent strip in the Micral- 24. All of the following may produce false-negative glucose
Test for microalbumin binds to: reactions except:
A. Protein A. Detergent contamination
B. Antihuman albumin antibody B. Ascorbic acid
C. Conjugated enzyme C. Unpreserved specimens
D. Galactoside D. Low urine temperature
16. All of the following are true for the ImmunoDip test for 25. The primary reason for performing a Clinitest is to:
microalbumin except: A. Check for high ascorbic acid levels
A. Unbound antibody migrates farther than bound B. Confirm a positive reagent strip glucose
antibody C. Check for newborn galactosuria
B. Blue latex particles are coated with antihuman D. Confirm a negative glucose reading
albumin antibody 26. The three intermediate products of fat metabolism
C. Bound antibody migrates further than unbound include
antibody all of the following except:
D. It utilizes an immunochromographic principle A. Acetoacetic acid
17. The principle of the protein-high pad on the Multistix B. Ketoacetic acid
Pro reagent strip is the: C. b -hydroxybutyric acid
A. Diazo reaction D. Acetone
B. Enzymatic dye-binding reaction 27. The most significant reagent strip test that is associated
C. Protein error of indicators with a positive ketone result is:
D. Microalbumin-Micral-Test A. Glucose
18. Which of the following is not tested on the Multistix Pro B. Protein
reagent strip? C. pH
A. Urobilinogen D. Specific gravity
B. Specific gravity 28. The primary reagent in the reagent strip test for ketones
C. Creatinine is:
D. Protein-high A. Glycine
94 Part Two | Urinalysis B. Lactose
19. The principle of the protein-low reagent pad on the C. Sodium hydroxide
Multistix D. Sodium nitroprusside
Pro is the: 29. Ketonuria may be caused by all of the following except:
A. Binding of albumin to sulphonphthalein dye A. Bacterial infections
B. Immunologic binding of albumin to antibody B. Diabetic acidosis
C. Reverse protein error of indicators reaction C. Starvation
D. Enzymatic reaction between albumin and dye D. Vomiting
20. The principle of the creatinine reagent pad on 30. Urinalysis on a patient with severe back and abdominal
microalbumin pain is frequently performed to check for:
reagent strips is the: A. Glucosuria
A. Double indicator reaction B. Proteinuria
B. Diazo reaction C. Hematuria
C. Pseudoperoxidase reaction D. Hemoglobinuria
D. Reduction of a chromogen 31. Place the appropriate number or numbers in front of
21. The purpose of performing an albumin:creatinine ratio each of the following statements. Use both numbers for
an answer if needed. Ehrlich reagent is:
1. Hemoglobinuria A. Positive reactions with porphobilinogen
2. Myoglobinuria B. Lack of specificity
A. ____ Associated with transfusion reactions C. Positive reactions with Ehrlich’s reactive substances
B. ____ Clear red urine and pale yellow plasma D. All of the above
C. ____ Clear red urine and red plasma 41. The reagent strip test for nitrite uses the:
D. ____ Associated with rhabdomyolysis A. Greiss reaction
E. ____ Produces hemosiderin granules in urinary B. Ehrlich reaction
sediments C. Peroxidase reaction
F. ____Associated with acute renal failure D. Pseudoperoxidase reaction
32. The principle of the reagent strip test for blood is based 42. All of the following can cause a negative nitrite reading
on the: except:
A. Binding of heme and a chromogenic dye A. Gram-positive bacteria
B. Peroxidase activity of heme B. Gram-negative bacteria
C. Reaction of peroxide and chromogen C. Random urine specimens
D. Diazo activity of heme D. Heavy bacterial infections
33. A speckled pattern on the blood pad of the reagent strip 43. A positive nitrite test and a negative leukocyte esterase
indicates: test is an indication of a:
A. Hematuria A. Dilute random specimen
B. Hemoglobinuria B. Specimen with lysed leukocytes
C. Myoglobinuria C. Vaginal yeast infection
D. All of the above D. Specimen older than 2 hours
Chapter 5 | Chemical Examination of Urine 95 44. All of the following can be detected by the leukocyte
34. List the following products of hemoglobin degradation esterase reaction except:
in the correct order by placing numbers 1 to 4 in the A. Neutrophils
blank. B. Eosinophils
A. ____Conjugated bilirubin C. Lymphocytes
B. ____Urobilinogen and stercobilinogen D. Basophils
C. ____Urobilin 45. Screening tests for urinary infection combine the
D. ____Unconjugated bilirubin leukocyte
35. The principle of the reagent strip test for bilirubin esterase test with the test for:
is the: A. pH
A. Diazo reaction B. Nitrite
B. Ehrlich reaction C. Protein
C. Greiss reaction D. Blood
D. Peroxidase reaction 46. The principle of the leukocyte esterase reagent strip test
36. An elevated urine bilirubin with a normal urobilinogen uses a:
is indicative of: A. Peroxidase reaction
A. Cirrhosis of the liver B. Double indicator reaction
B. Hemolytic disease C. Diazo reaction
C. Hepatitis D. Dye-binding technique
D. Biliary obstruction 47. The principle of the reagent strip test for specific gravity
37. The primary cause of a false-negative bilirubin reaction is: uses the dissociation constant of a(n):
A. Highly pigmented urine A. Diazonium salt
B. Specimen contamination B. Indicator dye
C. Specimen exposure to light C. Polyelectrolyte
D. Excess conjugated bilirubin D. Enzyme substrate
38. The purpose of the special mat supplied with the Ictotest 48. A specific gravity of 1.005 would produce the reagent
tablets is that: strip color:
A. Bilirubin remains on the surface of the mat. A. Blue
B. It contains the dye needed to produce color. B. Green
C. It removes interfering substances. C. Yellow
D. Bilirubin is absorbed into the mat. D. Red
39. The reagent in the Multistix reaction for urobilinogen is: 49. Reagent strip–specific gravity readings are affected by:
A. A diazonium salt A. Glucose
B. Tetramethylbenzidine B. Radiographic dye
C. p-Dimethylaminobenzaldehyde C. Alkaline urine
D. Hoesch reagent D. All of the above
40. The primary problem with urobilinogen tests using
Case Study with a burning sensation. She is a firm believer in the
1. A patient taken to the emergency department after an curative powers of vitamins. She has tripled her usual
episode of syncope has a fasting blood glucose level dosage of vitamins in an effort to alleviate her symptoms;
of 450 mg/dL. Results of the routine urinalysis are as however, the symptoms have persisted. She is
follows: given a sterile container and asked to collect a midstream
COLOR: Yellow KETONES: 2+ clean-catch urine specimen. Results of this
CLARITY: Clear BLOOD: Negative routine urinalysis are as follows:
SP. GRAVITY: 1.015 BILIRUBIN: Negative COLOR: Dark yellow KETONES: Negative
pH: 5.0 PROTEIN-LOW: 15 mg/dL CLARITY: Hazy BLOOD: Negative
PROTEIN-HIGH: NITRITE: Negative SP. GRAVITY: 1.012 BILIRUBIN: Negative
30 mg/dL pH: 7.0 UROBILINOGEN: Normal
GLUCOSE: 250 mg/dL LEUKOCYTES: Negative PROTEIN: Trace NITRITE: Negative
CREATININE: 200 mg/dL GLUCOSE: Negative LEUKOCYTES: 1+
a. Explain the correlation between the patient’s blood Microscopic
and urine glucose results. 8 TO 12 RBC/HPF Heavy bacteria
b. What is the most probable metabolic disorder associated 40 TO 50 WBC/HPF Moderate squamous epithelial
with this patient? cells
c. Considering the patient’s condition, what is the significance a. What discrepancies between the chemical and microscopic
of the patient’s protein to creatinine ratio test results are present? State and explain
result? a possible reason for each discrepancy.
d. If the patient in this study had a normal blood b. What additional chemical tests could be affected by
glucose level and normal protein and creatinine the patient’s vitamin dosage? Explain the principle
results, to what would the urinary glucose be of the interference.
attributed? c. Discuss the urine color and specific gravity results
2. Results of a urinalysis performed on a patient scheduled with regard to correlation and give a possible cause
for gallbladder surgery are as follows: for any discrepancy.
COLOR: Amber KETONES: Negative d. State three additional reasons not previously given
CLARITY: Hazy BLOOD: Negative for a negative nitrite test in the presence of increased
SP. GRAVITY: 1.022 BILIRUBIN: Moderate bacteria.
pH: 6.0 UROBILINOGEN: Normal 5. Results of a urinalysis collected following practice from
PROTEIN: Negative NITRITE: Negative a 20-year-old college athlete are as follows:
GLUCOSE: Negative LEUKOCYTES: Negative COLOR: Dark yellow KETONES: Negative
a. What would be observed if this specimen were CLARITY: Hazy BLOOD: 1+
shaken? SP. GRAVITY: 1.029 BILIRUBIN: Negative
b. Explain the correlation between the patient’s scheduled pH: 6.5 UROBILINOGEN: 1 EU
surgery and the normal urobilinogen. PROTEIN: 2+ NITRITE: Negative
c. If blood were drawn from this patient, how might GLUCOSE: Negative LEUKOCYTES: Negative
the appearance of the serum be described? The physician requests that the athlete collect another
d. What special handling is needed for serum and specimen in the morning prior to classes and practice.
urine specimens from this patient? a. What is the purpose of the second sample?
3. Results of a urinalysis on a very anemic and jaundiced b. What changes would you expect in the second
patient are as follows: sample?
COLOR: Red KETONES: Negative c. Is the proteinuria present in the first sample of prerenal,
CLARITY: Clear BLOOD: Large renal, or postrenal origin?
SP. GRAVITY: 1.020 BILIRUBIN: Negative 6. A construction worker is pinned under collapsed
pH: 6.0 UROBILINOGEN: 8 EU scaffolding
PROTEIN: Negative NITRITE: Negative for several hours prior to being taken to the
GLUCOSE: Negative LEUKOCYTES: Negative emergency room. His abdomen and upper legs are severely
a. Would these results be indicative of hematuria or bruised, but no fractures are detected. A specimen
hemoglobinuria? for urinalysis obtained by catheterization has the
b. Correlate the patient’s condition with the urobilinogen following results:
result. COLOR: Red-brown KETONES: Negative
c. Why is the urine bilirubin result negative in this CLARITY: Clear BLOOD: 4+
jaundiced patient? SP. GRAVITY: 1.017 BILIRUBIN: Negative
d. Would this method also measure urine porphyrins? pH: 6.5 UROBILINOGEN: 0.4 EU
Why or why not? PROTEIN: Trace NITRITE: Negative
4. A female patient arrives at the outpatient clinic with GLUCOSE: Negative LEUKOCYTES: Negative
symptoms of lower back pain and urinary frequency a. Would hematuria be suspected in this specimen?
Why or why not?
b. What is the most probable cause of the positive intensity in bright-field microscopy?
blood reaction? A. Centering screws
c. What is the source of the substance causing the positive B. Aperture diaphragm
blood reaction and the name of the condition? C. Rheostat
d. Would this patient be monitored for changes in D. Condenser aperture diaphragm
renal function? Why or why not? 8. Which of the following are reported as number per lpf?
7. Considering the correct procedures for care, technique, A. RBCs
and quality control for reagent strips, state a possible B. WBCs
cause for each of the following scenarios. C. Crystals
a. The urinalysis supervisor notices that an unusually D. Casts
large number of reagent strips are becoming discolored 9. The Sternheimer-Malbin stain is added to urine sediments
before the expiration date has been reached. to do all of the following except:
b. A physician’s office is consistently reporting positive A. Increase visibility of sediment constituents
nitrite test results with negative LE test results. B. Change the constituents’ refractive index
c. A student’s results for reagent strip blood and LE C. Decrease precipitation of crystals
are consistently lower than those of the laboratory D. Delineate constituent structures
staff. 10. Nuclear detail can be enhanced by:
d. One morning the urinalysis laboratory was reporting A. Prussian blue
results that were frequently questioned by B. Toluidine blue
physicians. C. Acetic acid
D. Both B and C
Part 6 11. Which of the following lipids is/are stained by Sudan III?
A. Cholesterol
1. Macroscopic screening of urine specimens is used to:
B. Neutral fats
A. Provide results as soon as possible
C. Triglycerides
B. Predict the type of urinary casts present
D. Both B and C
C. Increase cost-effectiveness of urinalysis
12. Which of the following lipids is/are capable of polarizing
D. Decrease the need for polarized microscopy
light?
2. Variations in the microscopic analysis of urine include all
A. Cholesterol
of the following except:
B. Neutral fats
A. Preparation of the urine sediment
C. Triglycerides
B. Amount of sediment analyzed
D. Both A and B
C. Method of reporting
13. The purpose of the Hansel stain is to identify:
D. Identification of formed elements
A. Neutrophils
3. All of the following can cause false-negative microscopic
B. Renal tubular cells
results except:
C. Eosinophils
A. Braking the centrifuge
D. Monocytes
B. Failing to mix the specimen
14. Crenated RBCs are seen in urine that is:
C. Dilute alkaline urine
A. Hyposthenuric
D. Using midstream clean-catch specimens
B. Hypersthenuric
4. The two factors that determine relative centrifugal force
C. Highly acidic
are:
D. Highly alkaline
A. Radius of rotor head and rpm
15. Differentiation among RBCs, yeast, and oil droplets may
B. Radius of rotor head and time of centrifugation
be accomplished by all of the following except:
C. Diameter of rotor head and rpm
A. Observation of budding in yeast cells
D. RPM and time of centrifugation
B. Increased refractility of oil droplets
5. When using the glass slide and cover-slip method, which of
C. Lysis of yeast cells by acetic acid
the following might be missed if the cover slip is overflowed?
D. Lysis of RBCs by acetic acid
A. Casts
16. A finding of dysmorphic RBCs is indicative of:
B. RBCs
A. Glomerular bleeding
C. WBCs
B. Renal calculi
D. Bacteria
C. Traumatic injury
6. Initial screening of the urine sediment is performed using
D. Coagulation disorders
an objective power of:
17. Leukocytes that stain pale blue with Sternheimer-Malbin
A. 4×
stain and exhibit brownian movement are:
B. 10×
A. Indicative of pyelonephritis
C. 40×
B. Basophils
D. 100×
C. Mononuclear leukocytes
7. Which of the following should be used to reduce light
D. Glitter cells
18. Mononuclear leukocytes are sometimes mistaken for: C. Contains neutral fats
A. Yeast cells D. Is contaminated with immersion oil
B. Squamous epithelial cells 29. The finding of yeast cells in the urine is commonly
C. Pollen grains associated
D. Renal tubular cells with:
19. When pyuria is detected in a urine sediment, the slide A. Cystitis
should be carefully checked for the presence of: B. Diabetes mellitus
A. RBCs C. Pyelonephritis
B. Bacteria D. Liver disorders
C. Hyaline casts Chapter 6 | Microscopic Examination of Urine 143
D. Mucus 30. The primary component of urinary mucus is:
20. Transitional epithelial cells are sloughed from the: A. Bence Jones protein
A. Collecting duct B. Microalbumin
B. Vagina C. Uromodulin
C. Bladder D. Orthostatic protein
D. Proximal convoluted tubule 31. The majority of casts are formed in the:
21. The largest cells in the urine sediment are: A. Proximal convoluted tubules
A. Squamous epithelial cells B. Ascending loop of Henle
B. Urothelial epithelial cells C. Distal convoluted tubules
C. Cuboidal epithelial cells D. Collecting ducts
D. Columnar epithelial cells 32. Cylindruria refers to the presence of:
22. A clinically significant squamous epithelial cell is the: A. Cylindrical renal tubular cells
A. Cuboidal cell B. Mucus-resembling casts
B. Clue cell C. Hyaline and waxy casts
C. Caudate cell D. All types of casts
D. Columnar cell 33. A person submitting a urine specimen following a
23. Forms of transitional epithelial cells include all of the strenuous
following except: exercise routine can normally have all of the following
A. Spherical in the sediment except:
B. Caudate A. Hyaline casts
C. Convoluted B. Granular casts
D. Polyhedral C. RBC casts
24. Increased transitional cells are indicative of: D. WBC casts
A. Catheterization 34. Prior to identifying an RBC cast, all of the following
B. Malignancy should be observed except:
C. Pyelonephritis A. Free-floating RBCs
D. Both A and B B. Intact RBCs in the cast
25. A primary characteristic used to identify renal tubular C. Presence of a cast matrix
epithelial cells is: D. A positive reagent strip blood reaction
A. Elongated structure 35. WBC casts are primarily associated with:
B. Centrally located nucleus A. Pyelonephritis
C. Spherical appearance B. Cystitis
D. Eccentrically located nucleus C. Glomerulonephritis
26. Following an episode of hemoglobinuria, RTE cells may D. Viral infections
contain: 36. The shape of the RTE cell associated with renal tubular
A. Bilirubin epithelial casts is primarily:
B. Hemosiderin granules A. Elongated
C. Porphobilinogen B. Cuboidal
D. Myoglobin C. Round
27. The predecessor of the oval fat body is the: D. Columnar
A. Histiocyte 37. When observing RTE casts, the cells are primarily:
B. Urothelial cell A. Embedded in a clear matrix
C. Monocyte B. Embedded in a granular matrix
D. Renal tubular cell C. Attached to the surface of a matrix
28. A structure believed to be an oval fat body produced a D. Stained by components of the urine filtrate
Maltese cross formation under polarized light but does not 38. The presence of fatty casts is associated with:
stain with Sudan III. The structure: A. Nephrotic syndrome
A. Contains cholesterol B. Crush injuries
B. Is not an oval fat body C. Diabetes mellitus
D. All of the above ____ Calcium phosphate 3. “Coffin lids”
39. Nonpathogenic granular casts contain: ____ Ammonium biurate 4. Dumbbell shape
A. Cellular lysosomes ____ Calcium carbonate 5. White precipitate
B. Degenerated cells 6. Thorny apple
C. Protein aggregates 49. Match the following abnormal crystals with their
D. Gram-positive cocci description/identifying characteristics:
40. All of the following are true about waxy casts except they: ____ Cystine 1. Bundles following refrigeration
A. Represent extreme urine stasis ____ Tyrosine 2. Highly alkaline pH
B. May have a brittle consistency ____ Cholesterol 3. Bright yellow clumps
C. Require staining to be visualized ____ Leucine 4. Hexagonal plates
D. Contain degenerated granules ____ Ampicillin 5. Flat plates, high specific gravity
41. Observation of broad casts represents: ____ Radiographic dye 6. Concentric circles, radial striations
A. Destruction of tubular walls ____ Bilirubin 7. Notched corners
B. Dehydration and high fever 8. Fine needles seen in liver disease
C. Formation in the collecting ducts 50. Match the following types of microscopy with their
D. Both A and C descriptions:
42. All of the following contribute to urinary crystals ____ Bright-field 1. Indirect light is reflected off the object
formation ____ Phase 2. Objects split light into two beams
except: ____ Polarized 3. Low refractive index objects may be
A. Protein concentration overlooked
B. pH ____ Dark-field 4. Three-dimensional images
C. Solute concentration ____Fluorescent 5. Forms halo of light around object
D. Temperature ____Interference 6. Detects electrons contrast emitted from
43. The most valuable initial aid for identifying crystals in a objects
urine specimen is: 7. Detects specific wavelengths
A. pH of light emitted from objects
B. Solubility Case Study
C. Staining 1. An 85-year-old woman with diabetes and a broken hip
D. Polarized microscopy has been confined to bed for the past 3 months. Results
44. Crystals associated with severe liver disease include all of of an ancillary blood glucose test are 250 mg/dL, and her
the following except: physician orders additional blood tests and a routine
A. Bilirubin urinalysis. The urinalysis report is as follows:
B. Leucine COLOR: Pale yellow KETONES: Negative
C. Cystine CLARITY: Hazy BLOOD: Moderate
D. Tyrosine SP. GRAVITY: 1.020 BILIRUBIN: Negative
45. All of the following crystals routinely polarize except: pH: 5.5 UROBILINOGEN: Normal
A. Uric acid PROTEIN: Trace NITRITE: Negative
B. Cholesterol GLUCOSE: 100 mg/dL LEUKOCYTES: 2+
C. Radiographic dye Microscopic:
D. Cystine 20 to 25 WBCs/hpf
144 Part Two | Urinalysis Many yeast cells and hyphae
46. Casts and fibers can usually be differentiated using: a. Why are yeast infections common in patients with
A. Solubility characteristics diabetes mellitus?
B. Patient history b. With a blood glucose level of 250 mg/dL, should
C. Polarized light glucose be present in the urine? Why or why not?
D. Fluorescent light c. Is there a discrepancy between the negative nitrite and
47. Match the following crystals seen in acidic urine with their the positive leukocyte esterase results? Explain your
description/identifying characteristics: answer.
____ Amorphous urates 1. Envelopes d. What is the major discrepancy between the chemical
____ Uric acid 2. Thin needles and microscopic results?
____ Calcium oxalate 3. Yellow-brown, e. Considering the patient’s history, what is the most
monohydrate whetstone probable cause for the discrepancy?
____ Calcium oxalate 4. Pink sediment 2. A medical technology student training in a newly renovated
dihydrate STAT laboratory is having difficulty performing a
5. Ovoid microscopic urinalysis. Reagent strip testing indicates the
48. Match the following crystals seen in alkaline urine with presence of moderate blood and leukocytes, but the student
their description/identifying characteristics: is also observing some large unusual objects resembling
____ Triple phosphate 1. Yellow granules crystals and possible casts. The student is also
____ Amorphous phosphate 2. Thin prisms
having difficulty keeping all of the constituents in focus d. What is the probable identification of the colorless
at the same time. crystals?
a. Why is the student having difficulty focusing? 5. A 2-year-old left unattended in the garage for 5 minutes
b. What is a possible cause of the unusual microscopic is suspected of ingesting antifreeze (ethylene glycol). The
constituents? urinalysis has a pH of 6.0 and is negative on the chemical
c. Should the student be concerned about the unusual examination. Two distinct forms of crystals are observed
microscopic constituents? Explain your answer. in the microscopic examination.
d. What microscopy technique could be used to aid in a. What type of crystals would you expect to be present?
differentiating a cast and an artifact? b. What is the other form of this crystal?
3. A prisoner sentenced to 10 years for selling illegal drugs c. Describe the two forms.
develops jaundice, lethargy, and hepatomegaly. d. Which form would you expect to be predominant?
A test for hepatitis B surface antigen is positive, and the 6. A female patient comes to the outpatient clinic with
patient is placed in the prison infirmary. When his condition symptoms of UTI. She brings a urine specimen with her.
appears to worsen and a low urinary output is observed, Results of the routine analysis performed on this specimen
the patient is transferred to a local hospital. are as follows:
Additional testing detects a superinfection with delta COLOR: Yellow KETONES: Negative
hepatitis virus and decreased renal concentrating ability. CLARITY: Hazy BLOOD: Small
Urinalysis results are as follows: SP. GRAVITY: 1.015 BILIRUBIN: Negative
COLOR: Amber KETONES: Negative pH: 9.0 UROBILINOGEN: Normal
CLARITY: Hazy BLOOD: Negative PROTEIN: Negative NITRITE: Negative
SP. GRAVITY: 1.011 BILIRUBIN: Large GLUCOSE: Negative LEUKOCYTE: 2+
pH: 7.0 UROBILINOGEN: 4.0 EU Microscopic:
PROTEIN: 2+ NITRITE: Negative 1 to 3 RBCs/hpf Heavy bacteria
GLUCOSE: Negative LEUKOCYTES: Negative 8 to 10 WBCs/hpf Moderate squamous
Microscopic: epithelial cells
2 to 4 WBCs/hpf 1 to 2 hyaline casts/lpf a. What discrepancies are present between the chemical
1 to 3 RBCs/hpf 1 to 2 granular casts/lpf and microscopic test results?
2 to 4 bile-stained RTE b. State a reason for the discrepancies.
cells/hpf c. Identify a chemical result in the urinalysis that confirms
0 to 1 RTE casts/lpf your reason for the discrepancies.
0 to 1 bile-stained waxy d. What course of action should the laboratory take to
casts/lpf obtain accurate results for this patient?
a. Based on the urinalysis results, in what area of the 7. A high school student is taken to the emergency room
nephron is damage occurring? with a broken leg that occurred during a football game.
b. Is this consistent with the patient’s primary diagnosis? The urinalysis results are as follows:
Explain your answer. COLOR: Dark yellow KETONES: Negative
c. What is causing the RTE cells to be bile stained? CLARITY: Hazy BLOOD: Moderate
d. Why is the urobilinogen level elevated? SP. GRAVITY: 1.030 BILIRUBIN: Negative
e. State a disorder in which the urobilinogen level would pH: 5.5 UROBILINOGEN: Normal
be elevated, but the bilirubin result would be negative. PROTEIN: 2+ NITRITE: Negative
4. A 30-year-old woman being treated for a UTI brings a GLUCOSE: Negative LEUKOCYTE: Negative
urine specimen to the Employee Health Clinic at 4:00 p.m. Microscopic:
The nurse on duty tells her that the specimen will be 0 to 3 WBCs/hpf
refrigerated 0 to 4 hyaline casts/lpf
and tested by the technologist the next morning. 0 to 3 granular casts/lpf
The technologist has difficulty interpreting the color of the Few squamous epithelial cells
reagent strip tests and reports only the following results: a. Are these results of clinical significance?
COLOR: Amber CLARITY: Slightly cloudy b. Explain the discrepancy between the chemical and
Microscopic: microscopic blood results.
3 to 5 RBCs/hpf c. What is the probable cause of the granular casts?
8 to 10 WBCs/hpf 8. As supervisor of the urinalysis section, you are reviewing
Moderate bacteria results. State why or why not each of the following results
Moderate colorless crystals appearing in bundles would concern you.
a. What could have caused the technologist to have a. The presence of waxy casts and a negative protein in
difficulty interpreting the reagent strip results? urine from a 6–month-old girl
b. Could this specimen produce a yellow foam when b. Increased transitional epithelial cells in a specimen
shaken? obtained following cystoscopy
c. What could the technologist do to aid in the identification c. Tyrosine crystals in a specimen with a negative
of the crystals? bilirubin test result
d. Cystine crystals in a specimen from a patient diagnosed C. Renal glycosuria
with gout D. Goodpasture’s syndrome
e. Cholesterol crystals in urine with a specific gravity 10. A disorder associated with polyuria and low specific
greater than 1.040 gravity is:
f. Trichomonas vaginalis in a male urine specimen A. Renal glucosuria
g. Amorphous urates and calcium carbonate crystals in B. Minimal change disease
a specimen with a pH of 7.0 C. Nephrogenic diabetes insipidus
D. Focal segmental glomerulosclerosis
Part 7 11. An inherited disorder producing a generalized defect in
tubular reabsorption is:
1. Most glomerular disorders are caused by:
A. Alport syndrome
A. Sudden drops in blood pressure
B. Acute interstitial nephritis
B. Immunologic disorders
C. Fanconi syndrome
C. Exposure to toxic substances
D. Renal glycosuria
D. Bacterial infections
12. A teenage boy who develops gout in his big toe and has a
2. Dysmorphic RBC casts would be a significant finding with
high serum uric acid should be monitored for:
all of the following except:
A. Fanconi syndrome
A. Goodpasture syndrome
B. Renal calculi
B. Acute glomerulonephritis
C. Uromodulin-associated kidney disease
C. Chronic pyelonephritis
D. Chronic interstitial nephritis
D. Henoch-Schönlein purpura
13. The only protein produced by the kidney is:
3. Occasional episodes of macroscopic hematuria over
A. Albumin
periods
B. Uromodulin
of 20 or more years are seen with:
C. Uroprotein
A. Crescentic glomerulonephritis
D. Globulin
B. IgA nephropathy
14. The presence of renal tubular epithelial cells and casts is
C. Nephrotic syndrome
an indication of:
D. Wegener granulomatosis
A. Acute interstitial nephritis
4. Antiglomerular basement membrane antibody is seen with:
B. Chronic glomerulonephritis
A. Wegener granulomatosis
C. Minimal change disease
B. IgA nephropathy
D. Acute tubular necrosis
C. Goodpasture syndrome
Study Questions
D. Diabetic nephropathy
Chapter 7 | Renal Disease 159
5. Antineutrophilic cytoplasmic antibody is diagnostic for:
15. Differentiation between cystitis and pyelonephritis is
A. IgA nephropathy
aided
B. Wegener granulomatosis
by the presence of:
C. Henoch-Schönlein purpura
A. WBC casts
D. Goodpasture syndrome
B. RBC casts
6. Respiratory and renal symptoms are associated with all of
C. Bacteria
the following except:
D. Granular casts
A. IgA nephropathy
16. The presence of WBCs and WBC casts with no bacteria is
B. Wegener granulomatosis
indicative of:
C. Henoch-Schönlein purpura
A. Chronic pyelonephritis
D. Goodpasture syndrome
B. Acute tubular necrosis
7. The presence of fatty casts is associated with all of the
C. Acute interstitial nephritis
following
D. Both B and C
except:
17. End-stage renal disease is characterized by all of the
A. Nephrotic syndrome
following except:
B. Focal segmental glomerulosclerosis
A. Hypersthenuria
C. Nephrogenic diabetes insipidus
B. Isosthenuria
D. Minimal change disease
C. Azotemia
8. The highest levels of proteinuria are seen with:
D. Electrolyte imbalance
A. Alport syndrome
18. Prerenal acute renal failure could be caused by:
B. Diabetic nephropathy
A. Massive hemorrhage
C. IgA nephropathy
B. Acute tubular necrosis
D. Nephrotic syndrome
C. Acute interstitial nephritis
9. Ischemia frequently produces:
D. Malignant tumors
A. Acute renal tubular necrosis
19. The most common composition of renal calculi is:
B. Minimal change disorder
A. Calcium oxalate
B. Magnesium ammonium phosphate Microscopic:
C. Cystine >100 RBCs/hpf 2–4 hyaline casts/lpf
D. Uric acid 8–10 WBCs/hpf 1–5 granular casts/lpf
20. Urinalysis on a patient with severe back pain being 0–2 waxy casts/lpf 0–2 broad waxy
evaluated a. What specific disease do the patient’s laboratory
for renal calculi would be most beneficial if it results and history suggest?
showed: b. Which laboratory result is most helpful in diagnosing
A. Heavy proteinuria this disease?
B. Low specific gravity c. What additional diagnosis does his current condition
C. Uric acid crystals suggest?
D. Microscopic hematuria d. What is the significance of the positive result for urine
Case Study glucose?
1. A 14-year-old boy who has recently recovered from a sore e. Is the specific gravity significant? Why or why not?
throat develops edema and hematuria. Significant laboratory f. What is the significance of the waxy casts?
results include a BUN of 30 mg/dL (normal 8 to 3. A 45-year-old woman is recovering from injuries received
23 mg/dL) and a positive group A streptococcal antibody in an automobile accident that resulted in her being taken
test. Results of a urinalysis are as follows: to the emergency department with severe hypotension.
Color: Red Ketones: Negative She develops massive edema. Significant laboratory results
Clarity: Cloudy Blood: Large include a BUN of 30 mg/dL (normal 8 to 23 mg/dL),
Sp. gravity: 1.020 Bilirubin: Negative cholesterol of 400 mg/dL (normal 150 to 240 mg/dL),
pH: 5.0 Urobilinogen: Normal triglycerides of 840 mg/dL (normal 10 to 190 mg/dL),
Protein: 3+ Nitrite: Negative serum protein of 4.5 mg/dL (normal 6.0 to 7.8 mg/dL),
Glucose: Negative Leukocyte: Trace albumin of 2.0 mg/dL (normal 3.2 to 4.5 mg/dL), and
Microscopic: a total urine protein of 3.8 g/d (normal 100 mg/d).
100 RBCs/hpf—many dysmorphic forms Urinalysis results are as follows:
5–8 WBCs/hpf Color: Yellow Ketones: Negative
0–2 granular casts/lpf Clarity: Cloudy Blood: Moderate
0–1 RBC casts/lpf Sp. gravity: 1.015 Bilirubin: Negative
a. What disorder do these results and history pH: 6.0 Urobilinogen: Normal
indicate? Protein: 4+ Nitrite: Negative
b. What specific characteristic was present in the Glucose: Negative Leukocyte: Negative
organism causing the sore throat? Microscopic:
c. What is the significance of the dysmorphic RBCs? 15–20 RBCs/hpf Moderate free fat droplets
d. Are the WBCs significant? Why or why not? 0–2 granular casts/lpf
e. What is the expected prognosis of this patient? 0–5 WBCs/hpf Moderate cholesterol
f. If the above urinalysis results were seen in a 5-year-old 0–2 fatty casts/lpf crystals
boy who has developed a red, patchy rash following 0–2 oval fat bodies/hpf
recovery from a respiratory infection, what disorder a. What renal disorder do these results suggest?
would you suspect? b. How does the patient’s history relate to this
2. B.J. is a seriously ill 40-year-old man with a history of disorder?
several episodes of macroscopic hematuria in the past c. What physiologic mechanism accounts for the
20 years. The episodes were associated with exercise or massive proteinuria?
stress. Until recently the macroscopic hematuria had d. What is the relationship of the proteinuria to the
spontaneously reverted to asymptomatic microscopic edema?
hematuria. Significant laboratory results include a BUN of e. What mechanism produces the oval fat bodies?
80 mg/dL (normal 8 to 23 mg/dL), serum creatinine of 4. A routinely active 4-year-old boy becomes increasingly
4.5 mg/dL (normal 0.6 to 1.2 mg/dL), creatinine clearance less active after receiving several preschool immunizations.
of 20 mL/min (normal 107 to 139 mL/min), serum calcium His pediatrician observes noticeable puffiness
of 8.0 mg/dL (normal 9.2 to 11.0 mg/dL), serum phosphorus around the eyes. A blood test shows normal BUN
of 6.0 mg/dL (normal 2.3 to 4.7 mg/dL), and an elevated and creatinine results and markedly decreased total
level of serum IgA. Results of a routine urinalysis protein and albumin values. Urinalysis results are as
are as follows: follows:
Color: Red Ketones: Negative Color: Yellow Ketones: Negative
Clarity: Slightly cloudy Blood: Large Clarity: Hazy Blood: Small
Sp. gravity: 1.010 Bilirubin: Negative Sp. gravity: 1.018 Bilirubin: Negative
pH: 6.5 Urobilinogen: Normal pH: 6.5 Urobilinogen: Normal
Protein: 300 mg/dL Nitrite: Negative Protein: 4+ Nitrite: Negative
Glucose: 250 mg/dL Leukocyte: Trace Glucose: Negative Leukocyte: Negative
Microscopic:
10–15 RBCs/hpf 0–1 hyaline casts/lpf d. Is this specimen suitable for the appearance of
0–4 WBCs/hpf 0–2 granular casts/lpf glitter cells? Explain your answer.
Moderate fat droplets 0–1 oval fat bodies/hpf e. What other population is at a high risk for
a. What disorder do the patient history, physical developing this condition?
appearance, and laboratory results suggest? f. What disorder might develop if this disorder is not
b. What other renal disorders produce similar treated?
urinalysis results? 7. A 10-year-old patient with a history of recurrent UTIs is
c. What is the expected prognosis for this patient? admitted to the hospital for diagnostic tests. Initial urinalysis
5. A 32-year-old construction worker experiences results are as follows:
respiratory difficulty followed by the appearance Color: Yellow Ketone: Negative
of blood-streaked sputum. He delays visiting a Clarity: Cloudy Blood: Small
physician until symptoms of extreme fatigue and Sp. gravity: 1.025 Bilirubin: Negative
red urine are present. A chest radiograph shows pulmonary pH: 8.0 Urobilinogen: Normal
infiltration, and sputum culture is negative Protein: 2+ Nitrite: Positive
for pathogens. Blood test results indicate anemia, Glucose: Negative Leukocyte: 2+
increased BUN and creatinine, and the presence Microscopic:
of antiglomerular basement membrane antibody. 6–10 RBCs/hpf 0–2 WBC casts/lpf Many
Urinalysis results are as follows: bacteria
Color: Red Ketones: Negative >100 WBCs/hpf 0–1 bacterial casts/lpf with
Clarity: Cloudy Blood: Large clumps
Sp. gravity: 1.015 Bilirubin: Negative A repeat urinalysis a day later has the following results:
ph: 6.0 Urobilinogen: Normal Color: Yellow Ketones: Negative
Protein: 3+ Nitrite: Negative clarity: Cloudy Blood: Small
Glucose: Negative Leukocyte: Trace Sp. gravity: >1.035 bilirubin: Negative
Microscopic: pH: 7.5 Urobilinogen: Normal
100 RBCs/hpf 0–3 hyaline casts/lpf Protein: 2+ Nitrite: Positive
10–15 WBCs/hpf 0–3 granular casts/lpf Glucose: Negative Leukocyte: 2+
0–2 RBCs casts/lpf Microscopic:
a. What disorder do the laboratory results suggest? 6–10 RBCs/hpf Many bacteria
b. How is this disorder affecting the glomerulus? 0–2 WBC casts/lpf
c. If the antiglomerular membrane antibody test is negative, >100 WBCs/hpf Moderate birefringent,
what disorder might be considered? 0–1 bacterial casts/lpf flat crystals
d. What is the diagnostic test for this disorder? a. What diagnostic procedure was performed on the patient
e. By what mechanism does this disorder affect the that could account for the differences in the two
glomerulus? urinalysis results?
6. A 25-year-old pregnant woman comes to the outpatient b. Considering the patient’s age and history, what is the
clinic with symptoms of lower back pain, urinary most probable diagnosis?
frequency, and a burning sensation when voiding. c. What microscopic constituent is most helpful to this
Chapter 7 | Renal Disease 161 diagnosis?
Her pregnancy has been normal up to this time. She d. What is the most probable cause of this disorder?
is given a sterile container and asked to collect a e. How can the presence of the bacterial cast be
midstream clean-catch urine specimen. Routine confirmed?
urinalysis results are as follows: f. What is the most probable source of the crystals
Color: Pale yellow Ketones: Negative present in the sediment?
Clarity: Hazy Blood: Small g. Without surgical intervention, what is the patient’s
Sp. gravity: 1.005 Bilirubin: Negative prognosis?
pH: 8.0 Urobilinogen: Normal 8. A 35-year-old patient being treated for a sinus infection
Protein: Trace Nitrite: Positive with methicillin develops fever, a skin rash, and edema.
Glucose: Negative Leukocyte: 2+ Urinalysis results are as follows:
Microscopic: Color: Dark yellow Ketones: Negative
6–10 RBCs/hpf Heavy bacteria Clarity: Cloudy Blood: Moderate
40–50 WBCs/hpf Moderate squamous Sp. gravity: 1.012 Bilirubin: Negative
epithelial cells pH: 6.0 Urobilinogen: Normal
a. What is the most probable diagnosis for this Protein: 3+ Nitrite: Negative
patient? Glucose: Negative Leukocyte: 2+
b. What is the correlation between the color and the Microscopic:
specific gravity? 20–30 RBCs/hpf 1–2 WBC casts/lpf
c. What is the significance of the blood and protein >100 WBCs/hpf 1–2 granular casts/lpf
tests? After receiving the urinalysis report, the physician
orders a test for urinary eosinophils. The urinary e. What changes will the patient be advised to make
eosinophil result is 10%. in his lifestyle to prevent future occurrences?
a. Is the urinary eosinophil result normal or
abnormal? Part 8
b. What is the probable diagnosis for this patient?
1. All states require newborn screening for PKU for early:
c. Discuss the significance of the increased WBCs
A. Modifications of diet
and WBC casts in the absence of bacteria.
B. Administration of antibiotics
d. How can this condition be corrected?
C. Detection of diabetes
9. Following surgery to correct a massive hemorrhage, a
D. Initiation of gene therapy
55-year-old patient exhibits oliguria and edema. Blood
2. All of the following disorders can be detected by newborn
test results indicate increasing azotemia and electrolyte
screening except:
imbalance. The glomerular filtration rate is 20 mL/min.
A. Tyrosyluria
Urinalysis results are as follows:
B. MSUD
Color: Yellow Ketones: Negative
C. Melanuria
Clarity: Cloudy Blood: Moderate
D. Galactosemia
Sp. gravity: 1.010 Bilirubin: Negative
3. The best specimen for early newborn screening is a:
162 Part Two | Urinalysis
A. Timed urine specimen
pH: 7.0 Urobilinogen: Normal
B. Blood specimen
Protein: 3+ Nitrite: Negative
C. First morning urine specimen
Glucose: 2+ Leukocyte: Negative
D. Fecal specimen
Microscopic:
4. Abnormal urine screening tests categorized as an overflow
50–60 RBCs/hpf 2–3 granular casts/lpf
disorder include all of the following except:
3–6 WBCs/hpf 2–3 RTE cell casts/lpf
A. Alkaptonuria
3–4 RTE cells/hpf 0–1 waxy casts/lpf
B. Galactosemia
0–1 broad granular casts/lpf
C. Melanuria
a. What diagnosis do the patient’s history and laboratory
D. Cystinuria
results suggest?
5. Which of the following disorders is not associated with the
b. What is the most probable cause of the patient’s
phenylalanine-tyrosine pathway?
disorder? Is this considered to be of prerenal, renal,
A. MSUD
or postrenal origin?
B. Alkaptonuria
c. What is the significance of the specific gravity
C. Albinism
result?
D. Tyrosinemia
d. What is the significance of the RTE cells?
6. The least serious form of tyrosylemia is:
e. State two possible reasons for the presence of the
A. Immature liver function
broad casts.
B. Type 1
10. A 40-year-old man develops severe back and abdominal
C. Type 2
pain after dinner. The pain subsides during the
D. Type 3
night but recurs in the morning, and he visits his
7. An overflow disorder of the phenylalanine-tyrosine
family physician. Results of a complete blood count
pathway
and an amylase are normal. Results of a routine
that would produce a positive reaction with the reagent
urinalysis are as follows:
strip test for ketones is:
Color: Dark yellow Ketones: Negative
A. Alkaptonuria
Clarity: Hazy Blood: Moderate
B. Melanuria
Sp. gravity: 1.030 Bilirubin: Negative
C. MSUD
pH: 5.0 Urobilinogen: Normal
D. Tyrosyluria
Protein: Trace Nitrite: Negative
8. An overflow disorder that could produce a false-positive
Glucose: Negative Leukocytes: Negative
reaction with Clinitest procedure is:
Microscopic
A. Cystinuria
15–20 RBCs/hpf
B. Alkaptonuria
0–2 WBCs/hpf
C. Indicanuria
Few squamous epithelial cells
D. Porphyrinuria
a. What condition could these urinalysis results and
9. A urine that turns black after sitting by the sink for
the patient’s symptoms represent?
several hours could be indicative of:
b. What would account for the crenated RBCs?
A. Alkaptonuria
c. Is there a correlation between the urine color and
B. MSUD
specific gravity and the patient’s symptoms?
C. Melanuria
d. Based on the primary substance that causes this
D. Both A and C
condition, what type of crystals might have been
10. Ketonuria in a newborn is an indication of:
present?
A. MSUD 20. Acetyl acetone is added to the urine before performing
B. Isovaleric acidemia the
C. Methylmalonic acidemia Ehrlich test when checking for:
D. All of the above A. Aminolevulinic acid
11. Urine from a newborn with MSUD will have a B. Porphobilinogen
significant: C. Uroporphyrin
A. Pale color D. Coproporphyrin
B. Yellow precipitate 21. The classic urine color associated with porphyria is:
C. Milky appearance A. Dark yellow
D. Sweet odor B. Indigo blue
12. Hartnup disease is a disorder associated with the C. Pink
metabolism D. Port wine
of: 22. Which of the following specimens can be used for
A. Organic acids porphyrin testing?
B. Tryptophan A. Urine
C. Cystine B. Blood
D. Phenylalanine C. Feces
13. 5-HIAA is a degradation product of: D. All of the above
A. Heme 23. The two stages of heme formation affected by lead
B. Indole poisoning are:
C. Serotonin A. Porphobilinogen and uroporphyrin
D. Melanin B. Aminolevulinic acid and porphobilinogen
14. Elevated urinary levels of 5-HIAA are associated with: C. Coproporphyrin and protoporphyrin
A. Carcinoid tumors D. Aminolevulinic acid and protoporphyrin
B. Hartnup disease 24. Hurler, Hunter, and Sanfilippo syndromes are hereditary
C. Cystinuria disorders affecting metabolism of:
D. Platelet disorders A. Porphyrins
Study Questions B. Purines
176 Part Two | Urinalysis C. Mucopolysaccharides
15. False-positive levels of 5-HIAA can be caused by a diet D. Tryptophan
high in: 25. Many uric acid crystals in a pediatric urine specimen may
A. Meat indicate:
B. Carbohydrates A. Hurler syndrome
C. Starch B. Lesch-Nyhan disease
D. Bananas C. Melituria
16. Place the appropriate letter in front of the following D. Sanfilippo syndrome
statements. 26. Deficiency of the GALT enzyme will produce a:
A. Cystinuria A. Positive Clinitest
B. Cystinosis B. Glycosuria
____ IEM C. Galactosemia
____ Inherited disorder of tubular reabsorption D. Both A and C
____ Fanconi syndrome 27. Match the metabolic urine disorders with their classic
____ Cystine deposits in the cornea urine abnormalities.
____ Early renal calculi formation ____ PKU A. Sulfur odor
17. Blue diaper syndrome is associated with: ____ Indicanuria B. Sweaty feet odor
A. Lesch-Nyhan syndrome ____ Cystinuria C. Orange sand in diaper
B. Phenylketonuria ____ Alkaptonuria D. Mousy odor
C. Cystinuria ____ Lesch-Nyhan disease E. Black color
D. Hartnup disease ____ Isovaleric acidemia F. Blue color
18. Homocystinuria is caused by failure to metabolize: Case Study
A. Lysine 1. A premature infant develops jaundice. Laboratory tests
B. Methionine are negative for hemolytic disease of the newborn, but
C. Arginine the infant’s bilirubin level continues to rise. Abnormal
D. Cystine urinalysis results include a dark yellow color, positive
19. The Ehrlich reaction will only detect the presence of: bilirubin, and needle-shaped crystals seen on microscopic
A. Uroporphyrin examination.
B. Porphobilinogen a. What is the most probable cause of the infant’s
C. Coproporphyrin jaundice?
D. Protoporphyrin
b. Could these same urine findings be associated with an b. Is there any correlation between the urinalysis results
adult? Explain your answer. and the substance observed in the child’s diapers?
c. What kinds of crystals are present? Name another type Explain your answer.
of crystal with a spherical shape that is associated with c. What disorder do the patient’s history and the urinalysis
this condition. results indicate?
d. When blood is drawn from this infant, what precaution d. Is the fact that this is a male patient of any significance?
should be taken to ensure the integrity of the Explain your answer.
specimen? e. Name the enzyme that is missing.
2. A newborn develops severe vomiting and symptoms 5. Shortly after arriving for the day shift in the urinalysis
of metabolic acidosis. Urinalysis results are positive for laboratory, a technician notices that an undiscarded urine
ketones and negative for glucose and other reducing has a black color. The previously completed
substances. report indicates the color to be yellow.
a. If the urine had an odor of “sweaty feet,” what metabolic a. Is this observation significant? Explain your answer.
disorder would be suspected? b. The original urinalysis report showed the specimen
b. If the newborn was producing dark brown urine with to be positive for ketones. Is this significant? Why or
a sweet odor, what disorder would be suspected? why not?
c. Would an MS/MS screen be helpful for the diagnosis? c. If the ketones are negative and the pH is 8.0 is this
3. A 13-year-old boy is awakened with severe back and significant? Why or why not?
abdominal pain and is taken to the emergency department 6. Bobby Williams, age 8, is admitted through the emergency
by his parents. A complete blood count is normal. department with a ruptured appendix. Although
Family history shows that both his father and uncle are surgery is successful, Bobby’s recovery is slow, and the
chronic kidney stone formers. Results of a urinalysis physicians are concerned about his health prior to the
are as follows: ruptured appendix. Bobby’s mother states that he has
COLOR: Yellow KETONES: Negative always been noticeably underweight despite eating a
APPEARANCE: Hazy BLOOD: Moderate balanced diet and having strong appetite and that his
SP. GRAVITY: 1.025 BILIRUBIN: Negative younger brother exhibits similar characteristics. A note
pH: 6.0 UROBILINOGEN: Normal in his chart from the first postoperative day reports that
PROTEIN: Negative NITRITE: Negative the evening nurse noticed a blue coloration in the urinary
GLUCOSE: Negative LEUKOCYTE: Negative catheter bag.
Microscopic: a. Is the catheter bag color significant?
>15–20 RBCs/hpf Few squamous epithelial b. What condition can be suspected from this history?
cells c. What is Bobby’s prognosis?
0–3 WBCs/hpf Many cystine crystals 7. An anemic patient is suspected of having lead poisoning.
a. What condition does the patient’s symptoms represent? a. What historical urine test was requested?
b. What is the physiologic abnormality causing this b. What should be added to the urine before testing?
condition? c. What element of heme synthesis would this be
c. If amino acid chromatography was performed on testing for?
this specimen, what additional amino acids could be d. Name another substance that can be tested for lead
present? poisoning.
d. Why are they not present in the microscopic e. What element of heme synthesis would this test for?
constituents?
e. Based on the family history, what genetic disorder Chapter 9
should be considered?
1. The functions of the CSF include all of the following
4. An 8–month-old boy is admitted to the pediatric unit
except:
with a general diagnosis of failure to thrive. The parents
A. Removing metabolic wastes
have observed slowness in the infant’s development of
B. Producing an ultrafiltrate of plasma
motor skills. They also mention the occasional appearance
C. Supplying nutrients to the CNS
of a substance resembling orange sand in the child’s
D. Protecting the brain and spinal cord
diapers. Urinalysis results are as follows:
2. The CSF flows through the:
COLOR: Yellow KETONES: Negative
A. Choroid plexus
APPEARANCE: Slightly BLOOD: Negative hazy
B. Pia mater
SP. GRAVITY: 1.024 BILIRUBIN: Negative
C. Arachnoid space
pH: 5.0 UROBILINOGEN: Normal
D. Dura mater
PROTEIN: Negative NITRITE: Negative
3. Substances present in the CSF are controlled by the:
GLUCOSE: Negative LEUKOCYTE: Negative
A. Arachnoid granulations
Microscopic:
B. Blood–brain barrier
Many uric acid crystals
C. Presence of one-way valves
a. Is the urine pH consistent with the appearance of uric
D. Blood–CSF barrier
acid crystals?
4. What department is the CSF tube labeled 3 routinely B. CNS malignancy
sent to? C. Multiple sclerosis
A. Hematology D. Hemorrhage
B. Chemistry 14. Neutrophils with pyknotic nuclei may be mistaken for:
C. Microbiology A. Lymphocytes
D. Serology B. Nucleated RBCs
5. The CSF tube that should be kept at room temperature is: C. Malignant cells
A. Tube 1 D. Spindle-shaped cells
B. Tube 2 15. The presence of which of the following cells is increased
C. Tube 3 in a parasitic infection?
D. Tube 4 A. Neutrophils
6. Place the appropriate letter in front of the statement that B. Macrophages
best describes CSF specimens in these two conditions: C. Eosinophils
A. Traumatic tap D. Lymphocytes
B. Intracranial hemorrhage 16. Macrophages appear in the CSF after:
____ Even distribution of blood in all tubes A. Hemorrhage
____ Xanthochromic supernatant B. Repeated spinal taps
____ Concentration of blood in tube 1 is greater C. Diagnostic procedures
than in tube 3 D. All of the above
____ Specimen contains clots 17. Nucleated RBCs are seen in the CSF as a result of:
7. The presence of xanthochromia can be caused by all of A. Elevated blood RBCs
the following except: B. Treatment of anemia
A. Immature liver function C. Severe hemorrhage
B. RBC degradation D. Bone marrow contamination
C. A recent hemorrhage 18. After a CNS diagnostic procedure, which of the following
D. Elevated CSF protein might be seen in the CSF?
8. A web-like pellicle in a refrigerated CSF specimen A. Choroidal cells
indicates: B. Ependymal cells
A. Tubercular meningitis C. Spindle-shaped cells
B. Multiple sclerosis D. All of the above
C. Primary CNS malignancy 200 Part Three | Other Body Fluids
D. Viral meningitis 19. Hemosiderin granules and hematoidin crystals are seen
9. Given the following information, calculate the CSF WBC in:
count: cells counted, 80; dilution, 1:10; large Neubauer A. Lymphocytes
squares counted, 10. B. Macrophages
A. 8 C. Ependymal cells
B. 80 D. Neutrophils
C. 800 20. Myeloblasts are seen in the CSF:
D. 8000 A. In bacterial infections
10. A CSF WBC count is diluted with: B. In conjunction with CNS malignancy
A. Distilled water C. After cerebral hemorrhage
B. Normal saline D. As a complication of acute leukemia
C. Acetic acid 21. Cells resembling large and small lymphocytes with
D. Hypotonic saline cleaved nuclei represent:
11. A total CSF cell count on a clear fluid should be: A. Lymphoma cells
A. Reported as normal B. Choroid cells
B. Not reported C. Melanoma cells
C. Diluted with normal saline D. Medulloblastoma cells
D. Counted undiluted 22. The reference range for CSF protein is:
12. The purpose of adding albumin to CSF before A. 6 to 8 g/dL
cytocentrifugation B. 15 to 45 g/dL
is to: C. 6 to 8 mg/dL
A. Increase the cell yield D. 15 to 45 mg/dL
B. Decrease the cellular distortion 23. CSF can be differentiated from serum by the presence of:
C. Improve the cellular staining A. Albumin
D. Both A and B B. Globulin
13. The primary concern when pleocytosis of neutrophils and C. Prealbumin
lymphocytes is found in the CSF is: D. Tau transferrin
A. Meningitis 24. In serum, the second most prevalent protein is IgG; in
CSF, the second most prevalent protein is: C. Glucose
A. Transferrin D. a -Ketoglutarate
B. Prealbumin Chapter 9 | Cerebrospinal Fluid 201
C. IgA 34. Before performing a Gram stain on CSF, the specimen
D. Ceruloplasmin must be:
25. Elevated CSF protein values can be caused by all of the A. Filtered
following except: B. Warmed to 37°C
A. Meningitis C. Centrifuged
B. Multiple sclerosis D. Mixed
C. Fluid leakage 35. All of the following statements are true about
D. CNS malignancy cryptococcal
26. The integrity of the blood–brain barrier is measured using meningitis except:
the: A. An India ink preparation is positive
A. CSF/serum albumin index B. A starburst pattern is seen on Gram stain
B. CSF/serum globulin ratio C. The WBC count is over 2000
C. CSF albumin index D. A confirmatory immunology test is available
D. CSF IgG index 36. The test of choice to detect neurosyphilis is the:
27. Given the following results, calculate the IgG index: CSF A. RPR
IgG, B. VDRL
50 mg/dL; serum IgG, 2 g/dL; CSF albumin, 70 mg/dL; C. FAB
serum albumin, 5 g/dL. D. FTA-ABS
A. 0.6 Case Study
B. 6.0 1. Three tubes of CSF containing evenly distributed visible
C. 1.8 blood are drawn from a 75-year-old disoriented patient
D. 2.8 and delivered to the laboratory. Initial test results are as
28. The CSF IgG index calculated in Study Question 27 follows:
indicates: WBC count: 250 m L Protein: 150 mg/dL
A. Synthesis of IgG in the CNS Glucose: 70 mg/dL Gram stain: No organisms
B. Damage to the blood–brain barrier seen
C. Cerebral hemorrhage Differential: Neutrophils, 68%; monocytes, 3%; lymphocytes,
D. Lymphoma infiltration 28%; eosinophils, 1%
29. The finding of oligoclonal bands in the CSF and not in Many macrophages containing ingested RBCs
the serum is seen with: a. What is the most probable condition indicated by
A. Multiple myeloma these results? State two reasons for your answer.
B. CNS malignancy b. Are the elevated WBC count and protein significant?
C. Multiple sclerosis Explain your answer.
D. Viral infections c. Are the percentages of the cells in the differential of
30. A CSF glucose of 15 mg/dL, WBC count of 5000, any significance? Explain your answer.
90% neutrophils, and protein of 80 mg/dL suggests: d. What two other structures besides RBCs might be
A. Fungal meningitis contained in the macrophages?
B. Viral meningitis e. If the blood was unevenly distributed and nucleated
C. Tubercular meningitis RBCs and capillary structures were seen instead of
D. Bacterial meningitis macrophages, what would this indicate?
31. A patient with a blood glucose of 120 mg/dL would have 2. A patient with AIDS is hospitalized with symptoms of
a normal CSF glucose of: high fever and rigidity of the neck. Routine laboratory
A. 20 mg/dL tests on the CSF show a WBC count of 100/m L with a
B. 60 mg/dL predominance of lymphocytes and monocytes, glucose
C. 80 mg/dL of 55 mg/dL (plasma: 85 mg/dL), and a protein of
D. 120 mg/dL 70 mg/dL. The Gram stain shows a questionable starburst
32. CSF lactate will be more consistently decreased in: pattern.
A. Bacterial meningitis a. What additional microscopic examination should be
B. Viral meningitis performed?
C. Fungal meningitis b. If the test is positive, what is the patient’s diagnosis?
D. Tubercular meningitis c. If the results of the test are questionable, what additional
33. Measurement of which of the following can be replaced testing can be performed?
by d. What could cause a false-positive reaction in this test?
CSF glutamine analysis in children with Reye syndrome? e. If the tests named in a and c are negative, the glucose
A. Ammonia level is 35 mg/dL, and a pellicle is observed in the
B. Lactate
fluid, what additional testing should be performed? than 50% of the corresponding blood glucose results
f. If CSF and serum IEF was performed on this patient, performed in the clinic.
what unusual findings might be present?
3. A 35-year-old woman is admitted to the hospital with Part 10
symptoms of intermittent blurred vision, weakness, and
1. Maturation of spermatozoa takes place in the:
loss of sensation in her legs. A lumbar puncture is performed
A. Sertoli cells
with the following results:
B. Seminiferous tubules
Appearance: Colorless, clear
C. Epididymis
WBC count: 35 cells/m L (90% lymphocytes)
D. Seminal vesicles
Glucose: 60 mg/dL (plasma: 100 mg/dL)
2. Enzymes for the coagulation and liquefaction of semen are
Protein: 60 mg/dL (serum: 8 g/dL)
produced by the:
Albumin: 40 mg/dL (serum: 6 g/dL)
A. Seminal vesicles
IgG globulin: 20 mg/dL (serum: 2 g/dL)
B. Bulbourethral glands
a. Name and perform the calculation to determine the
C. Ductus deferens
integrity of the patient’s blood–brain barrier.
D. Prostate gland
b. Does the patient have an intact barrier?
3. The major component of seminal fluid is:
c. Name and perform the calculation used to determine
A. Glucose
if IgG is being synthesized within the CNS.
B. Fructose
d. What does this result indicate?
C. Acid phosphatase
e. Considering the patient’s clinical symptoms and the
D. Citric acid
calculation results, what diagnosis is suggested?
4. If the first portion of a semen specimen is not collected,
f. If immunofixation electrophoresis is performed on
the
the patient’s serum and CSF, what findings would be
semen analysis will have which of the following?
expected?
A. Decreased pH
g. What substance in the CSF can be measured to monitor
B. Increased viscosity
this patient?
C. Decreased sperm count
Case Studies and Clinical Situations
D. Decreased sperm motility
202 Part Three | Other Body Fluids
5. Failure of laboratory personnel to document the time a
4. Mary Howard, age 5, is admitted to the pediatrics ward
semen sample is collected primarily affects the interpretation
with a temperature of 105%F, lethargy, and cervical rigidity.
of semen:
A lumbar spinal tap is performed, and three tubes of
A. Appearance
cloudy CSF are delivered to the laboratory. Preliminary
B. Volume
test results are as follows:
C. pH
Appearance: Cloudy
D. Viscosity
WBC count: 800 cells/m L
6. Liquefaction of a semen specimen should take place within:
Differential: 80% lymphocytes, 15% monocytes,
A. 1 hour
5% neutrophils
B. 2 hours
Protein: 65 mg/dL
C. 3 hours
Glucose: 70 mg/dL
D. 4 hours
Gram stain: No organisms seen
7. A semen specimen delivered to the laboratory in a condom
a. From these results, what preliminary diagnosis could
has a normal sperm count and markedly decreased sperm
the physician consider?
motility. This indicates:
b. Is the Gram stain result of particular significance?
A. Decreased fructose
Why or why not?
B. Antispermicide in the condom
c. Are the lymphocytes significant? Why or why not?
C. Increased semen viscosity
d. Would a CSF lactate test be of any value for the diagnosis?
D. Increased semen alkalinity
Why or why not?
8. An increased semen pH may be caused by:
5. State possible technical errors that could result in the
A. Prostatic infection
following
B. Decreased prostatic secretions
discrepancies:
C. Decreased bulbourethral gland secretions
a. An unusual number of Gram stains reported as
D. All of the above
grampositive
9. Proteolytic enzymes may be added to semen specimens to:
cocci fail to be confirmed by positive cultures.
A. Increase the viscosity
b. A physician complains that CSF differentials are being
B. Dilute the specimen
reported only as polynuclear and mononuclear cells.
C. Decrease the viscosity
c. Bacteria observed on the cytospin differential cannot
D. Neutralize the specimen
be confirmed by Gram stain or culture.
10. The normal sperm concentration is:
d. The majority of CSF specimens sent to the laboratory
A. Less than 20 million/m L
from the neurology clinic have glucose readings less
B. More than 20 million/mL B. Neckpiece
C. Less than 20 million/mL C. Midpiece
D. More than 20 million/m L D. Tail
11. Given the following information, calculate the sperm 20. The morphologic shape of a normal sperm head is:
concentration: A. Round
dilution, 1:20; sperm counted in five RBC B. Tapered
squares on each side of the hemocytometer, 80 and 86; C. Oval
volume, 3 mL. D. Amorphous
A. 80 million/mL 21. Normal sperm morphology when using the WHO criteria
B. 83 million/mL is:
C. 86 million/mL A. >30% normal forms
D. 169 million/m L B. <30% normal forms
12. Using the above information, calculate the sperm C. >15% abnormal forms
concentration D. <15% normal forms
when 80 sperm are counted in 1 WBC square and 22. Additional parameters measured by Kruger’s strict
86 sperm are counted in another WBC square. morphology
A. 83 million/mL include all of the following except:
B. 166 million per ejaculate A. Vitality
C. 16.6 million/mL B. Presence of vacuoles
D. 50 million per ejaculate C. Acrosome size
13. The primary reason to dilute a semen specimen before D. Tail length
performing a sperm concentration is to: 23. Round cells that are of concern and may be included in
A. Immobilize the sperm sperm counts and morphology analysis are:
B. Facilitate the chamber count A. Leukocytes
C. Decrease the viscosity B. Spermatids
D. Stain the sperm C. RBCs
14. When performing a sperm concentration, 60 sperm are D. Both A and B
counted in the RBC squares on one side of the 24. If 5 round cells per 100 sperm are counted in a sperm
hemocytometer morphology
and 90 sperm are counted in the RBC squares on smear and the sperm concentration is 30 million,
the other side. The specimen is diluted 1:20. The: the concentration of round cells is:
A. Specimen should be rediluted and counted A. 150,000
B. Sperm count is 75 million/mL B. 1.5 million
C. Sperm count is greater than 5 million/mL C. 300,000
D. Sperm concentration is abnormal D. 15 million
15. Sperm motility evaluations are performed: 25. Following an abnormal sperm motility test with a normal
A. Immediately after the specimen is collected sperm count, what additional test might be ordered?
B. Within 1 hour of collection A. Fructose level
C. After 3 hours of incubation B. Zinc level
D. At 6-hour intervals for 1 day C. MAR test
16. The percentage of sperm showing average motility that is D. Eosin-nigrosin stain
considered normal is: 26. Follow-up testing for a low sperm concentration would
A. 25% include testing for:
B. 50% A. Antisperm antibodies
C. 60% B. Seminal fluid fructose
D. 75% C. Sperm vitality
17. The purpose of the acrosomal cap is to: D. Prostatic acid phosphatase
A. Penetrate the ovum 27. The immunobead test for antisperm antibodies:
B. Protect the the nucleus A. Detects the presence of male antibodies
C. Create energy for tail movement B. Determines the presence of IgG, IgM, and IgA
D. Protect the neckpiece antibodies
18. The sperm part containing a mitochondrial sheath is the: C. Determines the location of antisperm antibodies
A. Head D. All of the above
B. Neckpiece 28. Measurement of a -glucosidase is performed to detect a
C. Midpiece disorder of the:
D. Tail A. Seminiferous tubules
19. All of the following are associated with sperm motility B. Epididymis
except the: C. Prostate gland
A. Head D. Bulbourethral glands
216 Part Three | Other Body Fluids observation? Why or why not?
29. A specimen delivered to the laboratory with a request b. State three chemical tests that would be of value in
for prostatic acid phosphatase and glycoprotein p30 was this analysis.
collected to determine: c. How does this abnormality affect fertility?
A. Prostatic infection 6. A specimen is delivered to the laboratory with a request to
B. Presence of antisperm antibodies determine if semen is present.
C. A possible rape a. What two chemical tests could be performed on the
D. Successful vasectomy specimen?
30. Following a negative postvasectomy wet preparation, the b. What additional examination could be performed on
specimen should be: the specimen?
A. Centrifuged and reexamined
B. Stained and reexamined Part 11
C. Reported as no sperm seen
1. The functions of synovial fluid include all of the following
D. Both A and B
except:
31. Standardization of procedures and reference values for
A. Lubrication for the joints
semen analysis is primarily provided by the:
B. Removal of cartilage debris
A. Manufacturers of instrumentation
C. Cushioning joints during jogging
B. WHO
D. Providing nutrients for cartilage
C. Manufacturers of control samples
2. The primary function of synoviocytes is to:
D. Clinical Laboratory Improvement Amendments
A. Provide nutrients for the joints
Case Study B. Secrete hyaluronic acid
1. A repeat semen analysis for fertility testing is reported as C. Regulate glucose filtration
follows: D. Prevent crystal formation
VOLUME: 3.5 mL SPERM COUNT: 6 million/mL 3. Which of the following is not a frequently performed test
VISCOSITY: Normal SPERM MOTILITY: 30%— on synovial fluid?
grade 1.0 A. Uric acid
pH: 7.5 MORPHOLOGY: <30% normal B. WBC count
forms—30 spermatids/100 sperm C. Crystal examination
The results correspond with the first analysis. D. Gram stain
a. List three abnormal parameters. 4. The procedure for collecting synovial fluid is called:
b. What is the sperm concentration? Is this normal? A. Synovialcentesis
c. What is the spermatid count? Is this normal? B. Arthrocentesis
d. Could the sperm concentration and the spermatid C. Joint puncture
count be related to the infertility? Explain your answer. D. Arteriocentesis
2. A semen analysis on a vasovasostomy patient has a normal 5. Match the following disorders with their appropriate
sperm concentration; however, motility is decreased, group:
and clumping is observed on the wet preparation. A. Noninflammatory
a. Explain the possible connection between these B. Inflammatory
observations C. Septic
and the patient’s recent surgery. D. Hemorrhagic
b. What tests could be performed to further evaluate the ____ Gout
patient’s infertility? ____ Neisseria gonorrhoeae infection
c. Briefly explain the different interpretations offered by ____ Systemic lupus erythematosus
these two tests. ____ Osteoarthritis
d. State three ways in which a positive result on these ____ Hemophilia
tests could be affecting male fertility. ____ Rheumatoid arthritis
3. A yellow-colored semen specimen is received in the ____ Heparin overdose
laboratory. 6. Normal synovial fluid resembles:
The analysis is normal except for decreased sperm A. Egg white
motility. Explain the possible connection between the two B. Normal serum
abnormal findings. C. Dilute urine
4. Abnormal results of a semen analysis are volume = 1.0 mL D. Lipemic serum
and sperm concentration = 1 million/mL. State a 7. Before testing, very viscous synovial fluid should be
nonpathologic treated with:
cause of these abnormal results. A. Normal saline
5. A semen specimen with normal initial appearance fails to B. Hyaluronidase
liquefy after 60 minutes. C. Distilled water
a. Would a specimen pH of 9.0 be consistent with this D. Hypotonic saline
8. Addition of a cloudy, yellow synovial fluid to acetic acid 18. Crystals that have the ability to polarize light are:
produces a/an: A. Corticosteroid
A. Yellow-white precipitate B. Monosodium urate
B. Easily dispersed clot C. Calcium oxalate
C. Solid clot D. All of the above
D. Opalescent appearance 19. In an examination of synovial fluid under compensated
9. Which of the following could be the most significantly polarized light, rhomboid-shaped crystals are observed.
affected if a synovial fluid is refrigerated before testing? What color would these crystals be when aligned parallel
A. Glucose to the slow vibration?
B. Crystal examination A. White
C. Mucin clot test B. Yellow
D. Differential C. Blue
10. The highest WBC count can be expected to be seen with: D. Red
A. Noninflammatory arthritis 20. If crystals shaped like needles are aligned perpendicular
B. Inflammatory arthritis to the slow vibration of compensated polarized light, what
C. Septic arthritis color are they?
D. Hemorrhagic arthritis A. White
11. When diluting a synovial fluid WBC count, all of the B. Yellow
following are acceptable except: C. Blue
A. Acetic acid D. Red
B. Isotonic saline 21. Negative birefringence occurs under red-compensated
C. Hypotonic saline polarized
D. Saline with saponin light when:
Study Questions A. Slow light is impeded more than fast light
226 Part Three | Other Body Fluids B. Slow light is less impeded than fast light
12. The lowest percentage of neutrophils would be seen in: C. Fast light runs against the molecular grain of the
A. Noninflammatory arthritis crystal
B. Inflammatory arthritis D. Both B and C
C. Septic arthritis 22. Synovial fluid cultures are often plated on chocolate agar
D. Hemorrhagic arthritis to detect the presence of:
13. All of the following are abnormal when seen in synovial A. Neisseria gonorrhoeae
fluid except: B. Staphylococcus agalactiae
A. Neutrophages C. Streptococcus viridans
B. Ragocytes D. Enterococcus faecalis
C. Synovial lining cells 23. The most frequently performed chemical test on synovial
D. Lipid droplets fluid is:
14. Synovial fluid crystals that occur as a result of purine A. Total protein
metabolism or chemotherapy for leukemia are: B. Uric acid
A. Monosodium urate C. Calcium
B. Cholesterol D. Glucose
C. Calcium pyrophosphate 24. Which of the following chemistry tests can be performed
D. Apatite on synovial fluid to determine the severity of RA?
15. Synovial fluid crystals associated with inflammation in A. Glucose
dialysis patients are: B. Protein
A. Calcium pyrophosphate dihydrate C. Lactate
B. Calcium oxalate D. Uric acid
C. Corticosteroid 25. Serologic tests on patients’ serum may be performed to
D. Monosodium urate detect antibodies causing arthritis for all of the following
16. Crystals associated with pseudogout are: disorders except:
A. Monosodium urate A. Pseudogout
B. Calcium pyrophosphate dihydrate B. Rheumatoid arthritis
C. Apatite C. Systemic lupus erythematosus
D. Corticosteroid D. Lyme arthritis
17. Synovial fluid for crystal examination should be examined Case Study
as a/an: 1. A 50-year-old man presents in the emergency department
A. Wet preparation with severe pain and swelling in the right knee.
B. Wright's stain Arthrocentesis
C. Gram stain is performed and 20 mL of milky synovial fluid is
D. Acid-fast stain
collected. The physician orders a Gram stain, culture, and C. Stored in the mesothelial cells
crystal examination of the fluid, as well as a serum uric D. Metabolized by the mesothelial cells
acid. She requests that the synovial fluid be saved for 4. Production of serous fluid is controlled by:
possible additional tests. A. Capillary oncotic pressure
a. Describe the tubes into which the fluid would be B. Capillary hydrostatic pressure
routinely placed. C. Capillary permeability
b. If the patient’s serum uric acid level is elevated, what D. All of the above
type of crystals and disorder are probable? 5. An increase in the amount of serous fluid is called a/an:
c. Describe the appearance of these crystals under direct A. Exudate
and compensated polarized light. B. Transudate
d. Why were the Gram stain and culture ordered? C. Effusion
2. A medical laboratory science student dilutes a synovial D. Malignancy
fluid sample before performing a WBC count. The fluid 6. Pleural fluid is collected by:
forms a clot. A. Pleurocentesis
a. Why did the clot form? B. Paracentesis
b. How can the student perform a correct dilution of the C. Pericentesis
fluid? D. Thoracentesis
c. After the correct dilution is made, the WBC count is 7. Place the appropriate letter in front of the following
100,000/_L. State two arthritis classifications that statements
could be considered. describing transudates and exudates.
d. State two additional tests that could be run to A. Transudate
determine the classification. B. Exudate
3. Fluid obtained from the knee of an obese 65-year-old ____ Caused by increased hydrostatic pressure
woman being evaluated for a possible knee replacement ____ Caused by increased capillary permeability
has the following results: ____ Caused by decreased oncotic pressure
APPEARANCE: Pale yellow and hazy ____ Caused by congestive heart failure
WBC COUNT: 500 cells/_L ____ Malignancy related
GRAM STAIN: Negative ____ Tuberculosis related
GLUCOSE: 110 mg/dL (serum glucose: 115 mg/dL) ____ Endocarditis related
a. What classification of joint disorder do these results ____ Clear appearance
suggest? 8. Fluid:serum protein and lactic dehydrogenase ratios are
b. Under electron microscopy, what crystals might be performed on serous fluids:
detected? A. When malignancy is suspected
c. How does the glucose result aid in the disorder B. To classify transudates and exudates
classification? C. To determine the type of serous fluid
4. A synovial fluid sample delivered to the laboratory for a D. When a traumatic tap has occurred
cell count is clotted. 9. Which of the following requires the most additional
a. What abnormal constituent is present in the fluid? testing?
b. What type of tube should be sent to the laboratory for A. Transudate
a cell count? B. Exudate
c. Could the original tube be used for a Gram stain and 10. An additional test performed on pleural fluid to classify
culture? Why or why not? the fluid as a transudate or exudate is the:
A. WBC count
Part 12 B. RBC count
1. The primary purpose of serous fluid is to: C. Fluid:cholesterol ratio
A. Remove waste products D. Fluid-to-serum protein gradient
B. Lower capillary pressure 11. A milky-appearing pleural fluid indicates:
C. Lubricate serous membranes A. Thoracic duct leakage
D. Nourish serous membranes B. Chronic inflammation
2. The membrane that lines the wall of a cavity is the: C. Microbial infection
A. Visceral D. Both A and B
B. Peritoneal 12. Which of the following best represents a hemothorax?
C. Pleural A. Blood HCT: 42 Fluid HCT: 15
D. Parietal B. Blood HCT: 42 Fluid HCT: 10
3. During normal production of serous fluid, the slight excess C. Blood HCT: 30 Fluid HCT: 10
of fluid is: D. Blood HCT: 30 Fluid HCT: 20
A. Absorbed by the lymphatic system 13. All of the following are normal cells seen in pleural fluid
B. Absorbed through the visceral capillaries except:
A. Mesothelial cells
B. Neutrophils D. Absolute lymphocyte count
C. Lymphocytes 242 Part Three | Other Body Fluids
D. Mesothelioma cells 24. Detection of the CA 125 tumor marker in peritoneal fluid
14. A differential observation of pleural fluid associated with indicates:
tuberculosis is: A. Colon cancer
A. Increased neutrophils B. Ovarian cancer
B. Decreased lymphocytes C. Gastric malignancy
C. Decreased mesothelial cells D. Prostate cancer
D. Increased mesothelial cells 25. Chemical tests primarily performed on peritoneal fluid
15. All of the following are characteristics of malignant cells include
except: all of the following except:
A. Cytoplasmic molding A. Lactose dehydrogenase
B. Absence of nucleoli B. Glucose
C. Mucin-containing vacuoles C. Alkaline phosphatase
D. Increased nucleus:cytoplasm ratio D. Amylase
16. A pleural fluid pH of 6.0 indicates: 26. Cultures of peritoneal fluid are incubated:
A. Esophageal rupture A. Aerobically
B. Mesothelioma B. Anaerobically
C. Malignancy C. At 37°C and 42°C
D. Rheumatoid effusion D. Both A and B
17. Plasma cells seen in pleural fluid indicate: Case Study
A. Bacterial endocarditis 1. Fluid from a patient with congestive heart failure is
B. Primary malignancy collected
C. Metastatic lung malignancy by thoracentesis and sent to the laboratory for
D. Tuberculosis infection testing. It appears clear and pale yellow and has a WBC
18. A significant cell found in pericardial or pleural fluid that count of 450/mL, fluid:serum protein ratio of 0.35, and
should be referred to cytology is a: fluid:serum LD ratio of 0.46.
A. Reactive lymphocyte a. What type of fluid was collected?
B. Mesothelioma cell b. Based on the laboratory results, would this fluid be
C. Monocyte considered a transudate or an exudate? Why?
D. Mesothelial cell c. List two other tests that could be performed to aid in
19. Another name for a peritoneal effusion is: classifying this fluid.
A. Peritonitis 2. A cloudy pleural fluid has a glucose level of 30 mg/dL
B. Lavage (serum glucose level is 100 mg/dL) and a pH of 6.8.
C. Ascites a. What condition do these results indicate?
D. Cirrhosis b. What additional treatment might the patient receive,
20. A test performed primarily on peritoneal lavage fluid is based on these results?
a/an: 3. The following results were obtained on a peritoneal fluid:
A. WBC count serum albumin, 2.8 g/dL; fluid albumin, 1.2 g/dL.
B. RBC count a. Calculate the SAAG.
C. Absolute neutrophil count b. Is this a transudate or an exudate? Why?
D. Amylase c. What is the most probable cause of the effusion?
21. The recommended test for determining whether 4. Paracentesis is performed on a patient with ascites. The
peritoneal fluid appears turbid and has an elevated WBC count.
fluid is a transudate or an exudate is the: Additional tests ordered include an absolute granulocyte
A. Fluid:serum albumin ratio count, amylase, creatinine, CEA, and CA 125.
B. Serum ascites albumin gradient a. What is the purpose for the absolute granulocyte
C. Fluid:serum lactic dehydrogenase ratio count? If it is less than 250 cells/mL, what condition
D. Absolute neutrophil count is indicated?
22. Given the following results, classify this peritoneal fluid: b. If the amylase level is elevated, what is its significance?
serum albumin, 2.2 g/dL; serum protein, 6.0 g/dL; fluid State an additional test that might be ordered.
albumin, 1.6 g/dL. c. Explain the significance of an elevated creatinine level.
A. Transudate d. What is the purpose of the CEA and CA 125 tests?
B. Exudate 5. Describe a situation in which paracentesis might be
23. Differentiation between bacterial peritonitis and cirrhosis performed on a patient who does not have ascites. If the
is done by performing a/an: RBC count is 300,000/mL, what does this indicate?
A. WBC count 6. Microscopic examination of an ascitic fluid shows many
B. Differential cells with nuclear and cytoplasmic irregularities containing
C. Absolute neutrophil count
psammoma bodies. The CEA test result is normal. B. Lecithin
What additional test would be helpful? C. Oxyhemoglobin
D. Sphingomyelin
Chapter 13 10. Plotting the amniotic fluid OD on a Liley graph represents
the severity of hemolytic disease of the newborn. A value
1. Which of the following is not a function of amniotic fluid?
that is plotted in zone II indicates what condition of the
A. Allows movement of the fetus
fetus?
B. Allows carbon dioxide and oxygen exchange
A. No hemolysis
C. Protects fetus from extreme temperature changes
B. Mildly affected fetus
D. Acts as a protective cushion for the fetus
C. Moderately affected fetus that requires close monitoring
2. What is the primary cause of the normal increase in
D. Severely affected fetus that requires intervention
amniotic
11. The presence of a fetal neural tube disorder may be
fluid as a pregnancy progresses?
detected by:
A. Fetal cell metabolism
A. Increased amniotic fluid bilirubin
B. Fetal swallowing
B. Increased maternal serum alpha-fetoprotein
C. Fetal urine
C. Decreased amniotic fluid phosphatidyl glycerol
D. Transfer of water across the placenta
D. Decreased maternal serum acetylcholinesterase
3. Which of the following is not a reason for decreased
12. True or False: An AFP MoM value greater than two times
amounts of amniotic fluid?
the median value is considered an indication of a neural
A. Fetal failure to begin swallowing
tube disorder.
B. Increased fetal swallowing
13. When severe HDN is present, which of the following tests
C. Membrane leakage
on the amniotic fluid would the physician not order to
D. Urinary tract defects
determine whether the fetal lungs are mature enough to
4. Why might a creatinine level be requested on an amniotic
withstand a premature delivery?
fluid?
A. AFP levels
A. Detect oligohydramnios
B. Foam stability index
B. Detect polyhydramnios
C. Lecithin/sphingomyelin ratio
C. Differentiate amniotic fluid from maternal urine
D. Phosphatidyl glycerol detection
D. Evaluate lung maturity
14. True or False: Prior to 35 weeks’ gestation, the normal L/S
5. Amniotic fluid specimens are placed in amber-colored
ratio is less than 1.6.
tubes
15. When performing an L/S ratio by thin-layer
prior to sending them to the laboratory to prevent the
chromatography,
destruction of:
a mature fetal lung will show:
A. Alpha-fetoprotein
A. Sphingomyelin twice as concentrated as lecithin
B. Bilirubin
B. No sphingomyelin
C. Cells for cytogenetics
C. Lecithin twice as concentrated as sphingomyelin
D. Lecithin
D. Equal concentrations of lecithin and sphingomyelin
6. How are specimens for FLM testing delivered to and stored
16. True or False: Phosphatidyl glycerol is present with an L/S
in the laboratory?
ratio of 1.1.
A. Delivered on ice and refrigerated
17. A rapid immunologic test for FLM that does not require
B. Immediately centrifuged
performance of thin-layer chromatography is:
C. Kept at room temperature
A. AFP levels
D. Delivered in a vacuum tube
B. Amniotic acetylcholinesterase
7. Why are amniotic specimens for cytogenetic analysis
C. Aminostat-FLM
incubated
D. Bilirubin scan
at 37°C prior to analysis?
18. Does the failure to produce bubbles in the Foam Stability
A. To detect the presence of meconium
Index indicate increased or decreased lecithin?
B. To differentiate amniotic fluid from urine
A. Increased
C. To prevent photo-oxidation of bilirubin to biliverdin
B. Decreased
D. To prolong fetal cell viability and integrity
19. The presence of phosphatidyl glycerol in amniotic fluid
8. Match the following colors in amniotic fluid with their
fetal lung maturity tests must be confirmed when:
significance.
A. Hemolytic disease of the newborn is present
___ A. Colorless 1. Fetal death
B. The mother has maternal diabetes
___ B. Dark green 2. Normal
C. Amniotic fluid is contaminated by hemoglobin
___ C. Red-brown 3. Presence of bilirubin
D. Neural tube disorder is suspected
___ D. Yellow 4. Presence of meconium
20. A lamellar body count of 50,000 correlates with:
9. A significant rise in the OD of amniotic fluid at 450 nm
A. Absent phosphatidyl glycerol and L/S ratio of 1.0
indicates the presence of which analyte?
B. L/S ratio of 1.5 and absent phosphatidyl glycerol
A. Bilirubin
C. OD at 650 nm of 1.010 and an L/S ratio of 1.1
D. OD at 650 nm of 0.150 and an L/S ratio of 2.0 C. Small intestine
Case Study D. Stomach
1. Amniocentesis is performed on a woman believed to be 3. Which of the following tests is not performed to detect
in approximately the 31st week of gestation. This is the osmotic diarrhea?
second pregnancy for this Rh-negative woman with diabetes. A. Clinitest
Spectrophotometric analysis of the fluid shows a B. Fecal fats
D A450 of 0.3. C. Fecal neutrophils
a. Based on the Liley graph, should the physician D. Muscle fibers
consider inducing labor? 4. The normal composition of feces includes all of the
b. What else must the physician consider prior to inducing following
labor? except:
The physician decides to induce labor based on a A. Bacteria
positive Aminostat-FLM. B. Blood
c. What information did this test provide for the physician? C. Electrolytes
d. Why did the physician prefer an Aminostat-FLM to an D. Water
L/S ratio in this situation? 5. What is the fecal test that requires a 3-day specimen?
2. Amniocentesis is performed following a maternal serum A. Fecal occult blood
AFP level of 2.2 MoM at 15 weeks’ gestation. B. APT test
a. What fetal condition is suspected? C. Elastase I
b. If the amniotic fluid AFP is 2.5 MoM, what additional D. Quantitative fecal fat testing
test could be performed? 6. The normal brown color of the feces is produced by:
c. In what situation would this additional test not be A. Cellulose
performed? B. Pancreatic enzymes
3. How might a dark green amniotic fluid affect the results C. Undigested foodstuffs
of the following tests? D. Urobilin
a. Foam Stability Index 7. Diarrhea can result from all of the following except:
b. L/S ratio A. Addition of pathogenic organisms to the normal
c. Aminostat-FLM intestinal flora
d. OD650 B. Disruption of the normal intestinal bacterial flora
4. How might a blood-streaked amniotic fluid affect the C. Increased concentration of fecal electrolytes
results of the following tests? D. Increased reabsorption of intestinal water and
a. L/S ratio electrolytes
b. AChE 8. Stools from persons with steatorrhea will contain excess
c. Bilirubin analysis amounts of:
d. Aminostat-FLM A. Barium sulfate
5. Amniocentesis is performed on a woman whose last two B. Blood
pregnancies resulted in stillbirths due to hemolytic disease C. Fat
of the newborn. A screening test performed at the hospital D. Mucus
is positive for bilirubin, and the specimen is sent to a 9. Which of the following pairings of stool appearance and
reference laboratory for a bilirubin scan. Physicians are cause does not match?
concerned when the report comes back negative. What A. Black, tarry: blood
factors would be considered in evaluating this result. B. Pale, frothy: steatorrhea
a. Correct specimen was sent C. Yellow-gray: bile duct obstruction
b. Specimen was refrigerated D. Yellow-green: barium sulfate
c. Specimen was exposed to light 10. Stool specimens that appear ribbon-like are indicative of
d. Specimen reached the reference lab within 30 mins which condition?
A. Bile-duct obstruction
B. Colitis
Chapter 14 C. Intestinal constriction
1. In what part of the digestive tract do pancreatic enzymes D. Malignancy
and bile salts contribute to digestion? 11. A black tarry stool is indicative of:
A. Large intestine A. Upper GI bleeding
B. Liver B. Lower GI bleeding
C. Small intestine C. Excess fat
D. Stomach D. Excess carbohydrates
2. Where does the reabsorption of water take place in the 12. Chemical screening tests performed on feces include all
primary digestive process? of the following except:
A. Large intestine A. APT test
B. Pancreas B. Clinitest
C. Pilocarpine iontophoresis 23. What is the recommended number of samples that
D. Quantitative fecal fats should
13. Secretory diarrhea is caused by: be tested to confirm a negative occult blood result?
A. Antibiotic administration A. One random specimen
B. Lactose intolerance B. Two samples taken from different parts of three
C. Celiac sprue stools
D. Vibrio cholerae C. Three samples taken from the outermost portion of
14. The fecal osmotic gap is elevated in which disorder? the stool
A. Dumping syndrome D. Three samples taken from different parts of two
B. Osmotic diarrhea stools
C. Secretory diarrhea 24. The immunochemical tests for occult blood:
D. Steatorrhea A. Test for human globulin
15. Microscopic examination of stools provides preliminary B. Give false-positive reactions with meat hemoglobin
information as to the cause of diarrhea because: C. Can give false-positive reactions with aspirin
A. Neutrophils are present in conditions caused by D. Are inhibited by porphyrin
toxin-producing bacteria 25. Guaiac tests for detecting occult blood rely on the:
B. Neutrophils are present in conditions that affect the A. Reaction of hemoglobin with hydrogen peroxide
intestinal wall B. Pseudoperoxidase activity of hemoglobin
C. Red and white blood cells are present if the cause is C. Reaction of hemoglobin with ortho-toluidine
bacterial D. Pseudoperoxidase activity of hydrogen peroxide
D. Neutrophils are present if the condition is of nonbacterial 26. What is the significance of an APT test that remains pink
etiology after addition of sodium hydroxide?
16. True or False: The presence of fecal neutrophils would be A. Fecal fat is present.
expected with diarrhea caused by a rotavirus. B. Fetal hemoglobin is present.
17. Large orange-red droplets seen on direct microscopic C. Fecal trypsin is present.
examination of stools mixed with Sudan III represent: D. Vitamin C is present.
A. Cholesterol 27. In the Van de Kamer method for quantitative fecal fat
B. Fatty acids determinations, fecal lipids are:
C. Neutral fats A. Converted to fatty acids prior to titrating with
D. Soaps sodium hydroxide
18. Microscopic examination of stools mixed with Sudan III B. Homogenized and titrated to a neutral endpoint with
and glacial acetic acid and then heated will show small sodium hydroxide
orange-red droplets that represent: C. Measured gravimetrically after washing
A. Fatty acids and soaps D. Measured by spectrophotometer after addition of
B. Fatty acids and neutral fats Sudan III
C. Fatty acids, soaps, and neutral fats 28. A patient whose stool exhibits increased fats, undigested
D. Soaps muscle fibers, and the inability to digest gelatin may
19. When performing a microscopic stool examination for have:
muscle fibers, the structures that should be counted: A. Bacterial dysentery
A. Are coiled and stain blue B. A duodenal ulcer
B. Contain no visible striations C. Cystic fibrosis
C. Have two-dimensional striations D. Lactose intolerance
D. Have vertical striations and stain red 268 Part Three | Other Body Fluids
20. A value of 85% fat retention would indicate: 29. A stool specimen collected from an infant with diarrhea
A. Dumping syndrome has a pH of 5.0. This result correlates with a:
B. Osmotic diarrhea A. Positive APT test
C. Secretory diarrhea B. Negative trypsin test
D. Steatorrhea C. Positive Clinitest
21. Which of the following tests would not be indicative of D. Negative occult blood test
steatorrhea? 30. Which of the following tests differentiates a
A. Fecal elastase-I malabsorption
B. Fecal occult blood cause from a maldigestion cause in steatorrhea?
C. Sudan III A. APT test
D. Van de Kamer B. D-xylose test
22. The term “occult” blood describes blood that: C. Lactose tolerance test
A. Is produced in the lower GI tract D. Occult blood test
B. Is produced in the upper GI tract Case Study
C. Is not visibly apparent in the stool specimen 1. Microscopic screening of a stool from a patient exhibiting
D. Produces a black, tarry stool
prolonged diarrhea shows increased fecal neutrophils and expected? Why?
normal qualitative fecal fats and meat fibers. c. The infant’s hemoglobin remains constant at 18 g/dL.
a. What type of diarrhea do these results suggest? What was the significance of the black stool?
b. Name an additional test that could provide more diagnostic d. Would this infant be expected to have ketonuria? Why
information. or why not?
c. Name one probable result for this test and one improbable
result. Chapter 15
d. If the test for fecal neutrophils were negative and the 1. Which of the following would not be a reason to collect a
fecal fat concentration increased, what type of diarrhea vaginal fluid for analysis?
would be suggested? A. Vaginitis
2. Laboratory studies are being performed on a 5-year-old B. Complications of pregnancy resulting in preterm
boy to determine whether there is a metabolic reason for delivery
his continued failure to gain weight. In addition to having C. Forensic testing in a sexual assault
blood drawn, the patient has a sweat chloride collected, D. Pregnancy testing
provides a random stool sample, and is asked to collect a 2. Which of the following organisms might not be detected
72-hour stool sample. if the specimen for vaginal secretion analysis had been
a. How can the presence of steatorrhea be screened for refrigerated?
by testing the random stool sample? A. Prevotella bivia
b. How does this test distinguish among neutral fats, B. Lactobacillus acidophilus
soaps, and fatty acids? C. Trichomonas vaginalis
c. What confirmatory test should be performed? D. Candida albicans
d. Describe the appearance of the stool specimens if 3. The appearance of the vaginal discharge in vulvovaginal
steatorrhea is present. candidiasis is described as:
e. If a diagnosis of cystic fibrosis is suspected, state two A. Clear and colorless
screening tests that could be performed on a stool B. Thin, homogeneous white-to-gray discharge
specimen to aid in the diagnosis. C. White, curd-like
f. State a possible reason for a false-negative reaction in D. Yellow-green and frothy
each of these tests. 4. A normal range for a vaginal pH is:
g. What confirmatory test could be performed? A. 3.8 to 4.5
3. A physician’s office laboratory is experiencing B. 5.0 to 6.0
inconsistencies C. 6.0 to 7.0
in the results of patient-collected specimens for D. 7.0 to 7.4
FOBT. Patients are instructed to submit samples from two 5. Which of the following tests differentiates budding yeast
areas of three different stools. Positive and negative controls cells from RBCs?
are producing satisfactory results. Patient #1 is a 30- A. pH
year-old woman taking over-the-counter medications for B. Saline wet mount
gastric reflux who has reported passing frequent, black C. KOH prep
stools. The results of all three specimens are negative for D. Whiff test
occult blood. Patient #2 is a 70-year-old woman suffering 6. Which of the following constituents is normal in healthy
from arthritis. She is taking the test as part of a routine vaginal fluid secretions?
physical. The results of all three specimens are positive A. Lactobacilli
for occult blood. Patient #3 is a 50-year-old man advised B. Basal cells
by the doctor to lose 30 lb. He has been doing well on a C. Trichomonas vaginalis
high-protein, low-carbohydrate diet. Two of his three D. Pseudohyphae
specimens are positive for occult blood. 7. Vaginal specimens collected for a saline wet prep should
a. What is the possible nonpathologic cause of the be:
unexpected A. Refrigerated to preserve motility
results for Patient #1? Patient #2? Patient #3? B. Prepared as soon as possible
b. How could the physician’s office staff avoid these C. Mailed to a reference laboratory
discrepancies? D. Preserved with potassium hydroxide
c. What testing methodology could be used for Patients #2 8. A positive amine (Whiff) test is observed in which of the
and #3? following syndromes?
4. A watery black stool from a neonate is received in the A. Bacterial vaginosis
laboratory B. Vulvovaginal candidiasis
with requests for an APT test, fecal pH, and a C. Atrophic vaginitis
Clinitest. D. Desquamative inflammatory vaginitis
a. Can all three tests be performed on this specimen? 9. A squamous epithelial cell covered with coccobacilli that
Why? extends beyond the cytoplasm margin is a:
b. If the Clinitest is positive, what pH reading can be A. Basal cell
B. Parabasal cell of vaginal itching and soreness. She indicated that
C. Clue cell she was experiencing increased vaginal secretions that were
d. Blastospore frothy and yellow to green. Upon examination, the healthcare
10. All of the following are diagnostic of bacterial vaginosis provider noted a strawberry-like cervix and performed
except: a pH test on the secretions. The pH was 5.5 and the wet
A. Vaginal pH of 3.8 prep demonstrated “swimming” organisms.
B. Presence of clue cells a. What is the probable diagnosis?
C. Positive amine (Whiff) test b. What other tests can be performed to confirm this
D. Thin, homogeneous white-to-gray vaginal diagnosis?
discharge c. What is the best course of treatment?
11. Which of the following organisms produces lactic acid d. Should her sexual partner be treated?
and hydrogen peroxide to maintain an acid vaginal e. List three complications that can occur with this
environment? disorder.
A. Gardnerella vaginalis 3. During a routine visit with the gynecologist, a 60-year-old
B. Mobiluncus spp. woman complained of vaginal dryness and soreness. During
C. Lactobacilli spp. the examination, the health-care provider noted erythema
D. b -Hemolytic streptococci of the vaginal mucosa. The pH of the vaginal secretions
12. All of the following are diagnostic of vulvovaginal was 6.0. The KOH and amine (Whiff) tests were negative.
candidiasis The microscopic examination revealed epithelial cells, basal
except: cells, decreased lactobacilli, and increased gram-positive
A. Large numbers of WBCs cocci and gram-negative rods.
B. Presence of clue cells a. What is the name of this condition?
C. Positive KOH test b. Explain how this condition can occur.
D. Vaginal pH of 4.0 c. What is the treatment for this condition?
13. All of the following are diagnostic of trichomoniasis
except:
A. Vaginal pH of 6.0
B. Positive amine test
C. Positive KOH test
D. Motile trichomonads present
14. The bacteria associated with desquamative inflammatory
vaginitis is:
A. b -Hemolytic streptococci
B. Trichomonas vaginalis
C. Gardnerella vaginalis
D. Mycoplasma hominis
15. The protein present in vaginal secretions that can identify
patients who are at risk for preterm delivery is:
A. Human chorionic gonadotropin
B. Estrogen
C. PAMG-1
D. Fetal fibronectin
Case Study
1. A 30-year-old woman has symptoms of dysuria, vaginal
itching, and a white, curd-like discharge. During her
visit at the Women’s Clinic, the patient revealed that she
had recently completed a regimen of broad-spectrum
antibiotics as treatment for a urinary tract infection. Her
health-care provider takes a swab of the vaginal secretions
for analysis.
a. What tests will be performed on the vaginal specimen?
b. Based on the patient history and observation of the
vaginal secretion, which test will be diagnostic for the
probable diagnosis?
c. What confirmatory test can be performed?
d. What is the probable diagnosis?
e. What is the first choice of treatment?
2. A sexually active teenager visited the Women’s Clinic
complaining

You might also like